You are on page 1of 75

Tuyển tập các bài toán và bài viết chủ đề

Olympic Toán học

Biên soạn: Trần Nam Dũng – Trịnh Thanh Đèo – Nguyễn Tiến Khải
Đơn vị tài trợ: Đại học FPT

Hà nội, Tp Hồ Chí Minh - 2007


Mục lục

1. Lời nói đầu


2. Bất đẳng thức Cauchy-Schwarz
3. Đề đề nghị cho cuộc thi giải toán của DDTH
4. Định lý Euler về tam giác pedal
5. Vai trò của các bài toán tổ hợp trong việc rèn luyện tư duy toán học và kỹ
năng giải toán
6. Olympic Toán Nhật Bản 2007
7. Olympic Toán Thái Lan 2006
8. Olympic Toán Thổ Nhĩ Kỳ 2007
9. Olympic Toán Estonia 2007
10. Olympic Toán Anh 2007
11. Olympic Toán Áo 2007
12. Olympic Toán Hàn Quốc 2007
13. Olympic Toán Iran 2007
14. Phục lục 1: Hướng dẫn làm bài thi vào Đại học FPT
15. Phụ lục 2: Dự thảo kế hoạch Cuộc thi Olympic Toán dành cho học sinh phổ
thông do đại học FPT tổ chức
16. Phụ lục 3: Cờ ca-rô, những điều chưa biết
Lời nói đầu

Kỳ thi Toán quốc tế lần thứ 48 tổ chức tại Việt Nam đã kết thúc tốt đẹp. Đoàn học
sinh Việt Nam chúng ta đã đạt thành tích cao nhất từ trước đến nay, BTC kỳ thi đã
hoàn thành xuất sắc nhiệm vụ, đem đến cho bạn bè thế giới một kỳ thi Olympic
đáng nhớ.

Là những người được tham dự vào công tác tổ chức IMO 2007, chúng tôi có điều
kiện tiếp xúc với các thầy cô giáo trưởng phó đoàn, các quan sát viên và về mặt
chuyên môn, đã học hỏi và thu nhận được rất nhiều qua những ngày tham gia IMO
2007.

Chúng tôi luôn nghĩ rằng, kiến thức, tài liệu khi được sẻ chia thì chúng sẽ được
nhân lên chứ không bị chia đi. Chính vì thế, được sự khuyến khích của GS
Nguyễn Văn Mậu, hiệu trưởng trường Đại học Khoa học Tự nhiên, người thầy
đáng kính của chúng tôi; được sự ủng hộ về nhiều mặt của Đại học FPT, đứng đầu
là PGS TS Trương Gia Bình và TS Hiệu trưởng Lê Trường Tùng, chúng tôi đã
hoàn thành tập sách này.

Tập sách cũng là sự mong mỏi của đông đảo các thầy cô giáo tham dự khoá Bồi
dưỡng giáo viên chuyên Toán tổ chức tại trường Đại học Khoa học Tự nhiên Hà
Nội từ 5-14/8. Sự mong mỏi đó cũng là một động lực lớn cho chúng tôi. Nhân đây,
chúng tôi xin bày tỏ lời cảm ơn sâu sắc đến GS Đặng Huy Ruận, người Thầy,
người Anh đáng kính và rất mực thân thiết của chúng tôi.

Chúng tôi cũng xin chân thành cảm ơn các cán bộ nhân viên Đại học FPT đã giúp
đỡ, ủng hộ để ấn bản này được sớm đến tay các thầy cô giáo. Đặc biệt tôi muốn
cảm ơn các anh Nguyễn Khắc Thành và Nguyễn Xuân Phong đã luôn ủng hộ
chúng tôi.

Chúng tôi cũng cảm ơn vợ, con và những người thân đã cảm thông, giúp đỡ và
đem lại nguồn cảm hứng cho công việc của chúng tôi.

Tập sách được hoàn thành trong khoảng thời gian kỷ lục: 3 ngày! Chính vì thế nó
không khỏi chứa những khiếm khuyết. Chúng tôi mong nhận được những ý kiến
đóng góp của quý thầy cô. Mọi góp ý xin gửi về địa chỉ: namdung@fpt.com.vn.

Tp HCM, 13-8-2007
Tập thể tác giả
Bất đẳng thức Cauchy-Schwarz

Gabriel Dospinescu, Trần Nam Dũng

Cùng với bất đẳng thức giữa trung bình cộng và trung bình nhân, BĐT Shur,
Jensen và Holder, bất đẳng thức Cauchy-Schwarz là một kết quả kinh điển, có
nhiều ứng dụng. Với cố gắng đưa ra một số khía cạnh của bất đẳng thức này,
chúng tôi sẽ tập trung vào sự đa dạng của các bài toán có thể giải bằng BĐT này.
Câu hỏi quan trọng nhất ở đây là: làm sao ta nhận biết được một bất đẳng thức có
thể giải bằng phương pháp này? Rất khó có thể nói một cách rõ ràng, nhưng có lẽ
là nên nghĩ đến BĐT Cauchy-Schwarz khi ta thấy tổng của các căn thức, tổng của
các bình phương hay đặc biệt là khi ta có các biểu thức chứa căn.
Đầu tiên, ta sẽ xét một số bài toán mà trong lời giải áp dụng BĐT Cauchy-
Schwarz ở dạng kinh điển:
2
 n   n 2  n 2 
  i i     ai   bi  .
a b
 i 1   i 1  i 1 
Khó khăn lớn nhất là chọn ai và bi . Chúng ta sẽ thấy rằng trong một vài
trường hợp, điều này là hiển nhiên, trong vài trường hợp khác lại không đơn giản
chút nào. Chúng ta giải một số bài toán:
Ví dụ 1
Cho x1 , x2 ,..., xn 1  0 thoả mãn điều kiện x1  x2  ...  xn  xn 1 . Chứng minh
rằng:
n n


i 1
xi ( xn 1  xi )  x
i 1
n 1 ( xn 1  xi ) .

Đề kiểm tra chọn ĐT Rumani, 1996


Lời giải:
Ngay cả một người chưa có nhiều kinh nghiệm cũng có thể áp dụng BĐT
Cauchy-Schwarz dưới dạng sao:
2
 n   n  n  n
  i n 1 i    i   n 1 i    xn 1 ( xn 1  xi ) .
x ( x  x )  x ( x  x )
 i 1   i 1  i 1  i 1
Sau ví dụ, đơn giản này, ta sẽ xem xét một bài toán khó hơn:
Ví dụ 2
Chứng minh rằng nếu x,y,z là các số thực thoả mãn điều kiện x 2  y 2  z 2  2 ,
thì bất đẳng thức sau đây đúng:
x  y  z  2  xyz .
Đề đề nghị IMO, Ba Lan
Lời giải:
Tại sao ta lại nghĩ đến BĐT Cauchy-Schwarz? Nguyên nhân rất rõ ràng: BĐT
cần chứng minh có dạng x(1  yz )  y  z  2 và ta cần phải đánh giá thông qua tổng
các bình phương: x 2  y 2  z 2 . Tuy nhiên, có rất nhiều cách để áp dụng Cauchy-
Schwarz. Chọn lựa ( x(1  yz )  y  z )2  ( x 2  y 2  z 2 )(2  (1  yz ) 2 ) không mấy thành
công. Vì thế có thể sẽ tốt hơn nếu coi y+z như một số hạng. Nếu chúng ta để ý
rằng bất đẳng thức có dầu bằng chẳng hạn khi x=1, y=1 và z=0, chọn lựa
x(1  yz )  y  z  ( x 2  ( y  z ) 2 )(1  (1  yz ) 2 ) trở nên khá tự nhiên. Như thế ta cần
chứng minh rằng 2(1  yz )(2  2 yz  y 2 z 2 )  4  y 3 z 3  y 2 z 2 , điều này hiển nhiên vì
2  y 2  z 2  2 yz .
Một ứng dụng không tầm thường khác của BĐT Cauchy-Schwarz là bài toán
sau.
Ví dụ 3
Cho a,b,c,x,y,z là các số thực dương thoả mãn điều kiện ax  by  cz  xyz .
Chứng minh rằng:
x y z  ab  bc  ca .
Lời giải:
Trong ví dụ này, sự xuất hiện căn thức ở vế phải cho chúng ta dấu hiệu rằng
BĐT Cauchy-Schwarz có thể có hiệu quả. Sẽ rất tuyệt nếu ta có
( x  y  z ) 2  6(a  b  c ) , nhưng điều này ở đây không hằng đúng. Cũng có thể thấy
xyz không đóng vai trò quan trọng trong bài toán này, vì vậy, tốt nhất là loại bỏ nó.
a b c
Vì vậy, ta viết    1 và bây giờ thì phép thế a=yzu, b=zxv và c=xyw trở
yz zx xy
nên tự nhiên.
Như vậy, ta cần chứng minh rằng
z ( yu  xv)  x( zv  yw)  y ( zu  xw)  x  y  z với u+v+w=1. Bây giờ thì ta có
thể thấy dạng của BĐT Cauchy-Schwarz:
 
2
z ( yu  xv)  x( zv  yw)  y ( zu  xw)  ( x  y  z )( yu  yw  xv  xw  zu  zv) và
biểu thức cuối cùng nhỏ hơn ( x  y  z ) 2 vì u+v+w=1.
Ta thấy rằng ta có thể áp dụng Cauchy-Schwarz khi ta có các tổng. Thế với tích
số thì sao? Ví dụ sau đây sẽ chứng tỏ rằng trong trường hợp này, ta cần có 1 chút
sáng tạo:
Ví dụ 4
Cho n>1 là số nguyên và a1 ,..., an là các số thực dương. Chứng minh bất đẳng
thức:
(a13  1)...( an3  1)  (a12 a2  1)...(an2 a1  1) .
Cuộc thi CH Séc-Slovakia-Ba Lan, 2001
Lời giải:
Ta sẽ thử áp dụng Cauchy-Schwarz cho mỗi thừa số có mặt ở vế phải. Một cách
tự nhiên, ta viết (1  a12 a2 ) 2  (1  a13 )(1  a22 a1 ) , vì ta cần 1  a13 , biểu thức có mặt ở vế
trái. Tương tự, ta có thể viết (1  a22 a3 ) 2  (1  a23 )(1  a32 a2 ) ,…,
(1  an2 a1 ) 2  (1  an3 )(1  a12 an ) .
Nhân các bất đẳng thức này với nhau, ta được
 (a a  1)...(an2 a1  1)   (a13  1)...(an3  1)(1  a22 a1 )...(1  a12 an ) (*).
2 2
1 2

Nhưng đây chưa phải là điều ta cần. Như thế Cauchy không có hiệu quả cho BĐT
này. Không hẳn thế! Ta lại áp dụng lý luận này một lần nữa để có that
 (1  a22 a1 )...(1  a12 an )   (a13  1)...(an3  1)(a12a2  1)...(an2a1  1) (**).
2

Như thế nếu (a12 a2  1)...(an2 a1  1)  (1  a22 a1 )...(1  a12 an ) , (*) sẽ cho chúng ta kết quả,
còn nếu ngược lại thì (**) sẽ kết thúc lời giải của chúng ta.
Bây giờ ta sẽ xem xét những bài toán khó hơn.
Ví dụ 5
Cho x,y>0 sao cho x 2  y 3  x3  y 4 . Chứng minh rằng x3  y 3  2 .
Nga, 1999
Lời giải:
Ý tưởng ở đây là chặn x3  y 3 bởi A( x3  y 4 ) với A nào đó. Điều này là có lý
nếu nhìn vào các luỹ thừa. Ta thử áp dụng vài thủ thuật với Cauchy-Schwarz và
bất đẳng thức trung bình cộng, trung bình nhân:
2
 x2  y 2  x3  y3 
( x  y )  ( x  y )( x  y )  ( x  y )( x  y )  
3 3 2 3 4 3 2 2 3 3 2
 . Như vậy ta đã
 2 
chứng tỏ rằng x  y  x  y . Nhưng ( x  y )  ( x  y )( x  y ) và do đó
3 3 2 2 2 2 2 3 3

x 2  y 2  x  y  2( x 2  y 2 ) từ đây ta suy ra x  y  2 và x  y  2 .
2 2 3 3

Một áp dụng đẹp của BĐT Cauchy-Schwarz là ví dụ sau:


Ví dụ 6
Cho a,b,c,x,y,z là các số thực sao cho (a  b  c)( x  y  z )  3 và
(a  b 2  c 2 )( x 2  y 2  z 2 )  4 . Chứng minh rằng ax+by+cz  0 .
2

Cuộc thi Mathlinks, 2005


Lời giải:
Một lần nữa có vẻ như là có thể áp dụng trực tiếp Cauchy-Schwarz. Thế nhưng
chúng ta sẽ thấy rằng chúng ta sẽ gặp khó khăn khi làm điều đó. Phải áp dụng
Cauchy-Schwarz ở đây thế nào? Câu trả lời đến từ một câu hỏi khác: ax+by+cz  0
nghĩa là gì? Điều này tương đương với (ax  by  cz  t ) 2  (ax  by  cz) 2  t 2 với số
thực dương t nào đó. Vì thế, sẽ rất tuyệt nếu ta tìm được t như thế. Điều này dẫn
đến việc đánh giá biểu thức ax+by+cz-t. Cố gắng thuần nhất hoá các biểu thức, ta
có dãy các hệ thức và bất đẳng thức sau:
2 2
 t   t 
ax  by  cz  t   ax  by  cz  ( a  b  c )( x  y  z )     a  ( x  y  z )  x  
2

 3   3 
 t  
2

   a 2      ( x  y  z )  x   . Như vậy, ta cần tìm t sao cho


 3  

2
4 t 
2 
 ( x  y  z )  x   t 2 (*). Bằng cách này, ta sẽ có
x y z
2 2
3 
(ax  by  cz  t )  (ax  by  cz )  t 2 và lời giải kết thúc. Nhưng (*) tương đương
2 2

t (t  2) t2 2
với ( x  y  z )  x  y  z  ( x  y 2  z 2 ) . Rõ ràng chọn lựa tốt nhất là
2 2 2 2

3 4
t=2. Như vậy, một cách gián tiếp ta chứng minh được rằng | 2  ax  by  cz | 2 và
từ điều này suy ra ax+by+cz  0 .
Đã lâu ta không thấy căn thức, đây là thời điểm để giải hai bài toán như vậy:
Ví dụ 7
Chứng minh rằng nếu x,y,z  [1,1] thoả mãn điều kiện x+y+z+xyz=0, thì ta có:
x 1  y 1  z 1  3 .
Lời giải:
Ta có căn thức, vì vậy đầu tiên ta sẽ áp dụng Cauchy ở dạng hiển nhiên:
x  1  y  1  z  1  3( x  y  z  3) .
Nhưng biểu thức vế phải có nhỏ hơn 3? Có, nếu như x  y  z  0 . Bây giờ ta
giả sử rằng x+y+z > 0. Khi đó xyz<0. Có thể giả sử z<0. Từ đây ta có x, y  (0,1] .
Ta sẽ không từ bỏ và cố gắng áp dụng Cauchy-Schwarz một lần nữa, như lần này
là cho 2 căn:
x 1  y 1  z  1  2x  2 y  4  z 1 .
Và ta cần chứng minh rằng
2( x  y ) z  z ( xy  1) z
2x  2 y  4  z 1  3    
2  2x  2 y  4 1 z 1 2  2x  2 y  4 1  z  1
(1  x )(1  y )
 2 xy  2(1  xy ) 1  z  2 x  2 y  4 . Vì 1  z  , tất cả quy về việc
1  xy
x y
chức minh rằng xy  (1  x)(1  y )(1  xy )  1  . Ta muốn sử dụng Cauchy
2
thế nào đó để làm mất đi 1-x ở vế trái. Cụ thể là:
x y
xy  (1  x)(1  y )(1  xy )  x  xy 2  1  x  1  xy  y  xy 2  1  xy  y  1  1 
2
Và bài toán được giải quyết hoàn toàn.
Có thể ví dụ khó nhất trong các bài toán dạng này là bài toán sau:
Ví dụ 8
Chứng minh rằng với mọi a,b,c,x,y,z>0 ta có bất đẳng thức:
a b c xy  yz  zx
( y  z)  ( z  x)  ( x  y)  3  .
bc ca ab x yz
Crux Mathematicorum
Lời giải:
Ở đây chẳng thấy có dấu hiệu gì của Cauchy. Nhưng thực tế thì có ít nhất hai lời
giải của bài toán này sử dụng Cauchy-Schwarz.
Lời giải 1:
 b  c ( y  z )   ( y  z )    a   b  c   2   (b  c)  b  c   2  
a  y z 1  yz 1 2
yz

Ta sẽ chứng minh rằng


3 yz
1
 ( y  z)  2
  2 x
2 x
(1)
Từ đây suy ra điều phải chứng minh. (1) tương đương với
 ( x  
x 2  xy  yz  zx )   x   3  yz   2  x 
2

Bởi vì
 x 2  ( xy  yz  zx)   x 2
 9  yz 
Nên ta chỉ cần chứng minh
 x  x 2
 9  yz     x   3  yz 
2

Nhưng điều này là hiển nhiên sau khi bình phương hai vế của BĐT.

Lời giải 2:
Từ tính thuần nhất đối với x,y,z ta có thể giả sử rằng x+y+z=1. Bất đẳng thức
trở thành:
a b c a
3( xy  yz  zx)  x y z .
bc ca ab bc
Bây giờ ta bắt đầu áp dụng Cauchy-Schwarz. Cách tự nhiên nhất là
2
a b c  a 
bc
x
c a
y
ab
z  x2  y 2  z 2    b  c  . Bây giờ ta cố gắng sử dụng
3(xy+yz+zx) vì vậy ta có thể bổ sung (sử dụng Cauchy-Schwarz) 2(xy+yz+zx)
vào với x 2  y 2  z 2 . Như thế ta cần một điều kiểu như
2
 a 
 xy  A   xy  B   x    b  c  2

2
 a 
  x  2 xy  A  B   
2
 .
bc
Ta cũng phải có A  B  3 xy  yz  zx và A+B cần phải càng nhỏ các tốt.
9 3
Vì thế lựa chọn duy nhất là A  B  ( xy  yz  zx)  và ta chỉ còn cần chứng
4 4
minh rằng
2
3  a  a ab 3
2       . Nhưng đây là một bài toán đơn
4 bc bc (c  a)( c  b) 4
giản, vì nó có thể biến đổi thành  a(b  c) 2
 0.
Bây giờ ta sẽ nói về một thủ thuật đã được áp dụng khá nhiều trong những năm
gần đây để giải các bài toán thi. Đây chính là một trong những phương án của
BĐT Cauchy-Schwarz:
2
 n 
n 2   ak 
ak  k 1 
 b
 n , với mọi ai thực và bi dương.
k 1 k
 bk
k 1

Một áp dụng đơn giản của thủ thuật này là bài toán sau, được đề nghị tại Cuộc
thi giữa các thành phố (Tournament of The Towns competition)
Ví dụ 9
Chứng minh rằng với mọi các số thực dương a,b,c ta có bất đẳng thức sau:
a3 b3 c3 a2  b2  c2
   .
a 2  ab  b 2 b2  bc  c 2 c 2  ac  a 2 abc
Lời giải:
a  b2  c2 
2 2

Nếu ta viết lại vế phải dưới dạng , ta sẽ biết ta cần làm gì:
(a  b  c)  a 2  b 2  c 2 

 a2    a2 
2 2
a3
 a 2  ab  b2   a(b2  ab  a 2 )  a(a 2  ab  b2 ) .

Như vậy, ta sẽ có ngay điều phải chứng minh nếu
 a  a 2  ab  b2     a     a 2  , và thực tế thì ở đây xảy ra dấu bằng.
Có thể có cảm nhận rằng bất đẳng thức chứa phân số nào cũng có thể tấn công
bằng cách này. Điều này hoàn toàn sai lầm! Sẽ có những trường hợp mà gần như
ta không thể tìm được ai và bi . Ta sẽ thảo luận một số bài toán mà trong đó thủ
thuật này được áp dụng một cách không hiển nhiên chút nào.
Ví dụ 10
Cho a1 , a2 ,..., an  0 thoả mãn điều hiện a1  a2  ...  an  1 . Chứng minh rằng:
 a1 a  n
 a1a2  a2 a3  ...  an a1    ...  2 n   .
 a  a2 a1  a1  n  1
2
2

Lời giải:
Câu hỏi chính ở đây là: từ đâu ta có được a1a2  a2 a3  ...  an a1 ? Có thể biểu thức
a12 an2
này sẽ đến từ  ...  sau khi ta áp dụng thủ thuật. Vì thế, ta hãy thử làm
a1a2 an a1
điều này (mặc dù hoàn toàn không hiển nhiên là điều này sẽ dẫn đến lời giải):
a1 a2 a a2 a2 1
  ...  n  1  ...  n  .
a2 a3 a1 a1a2 an a1 a1a2  ...  an a1
a1 a n  a1 an 
Bài toán mới, chứng minh rằng  ...  2 n    ...   trông có vẻ
a  a2
2
2 a1  a1 n  1  a2 a1 
còn khó hơn, nhưng chúng ta sẽ thấy rằng chúng ta phải tiến thêm một bước để
a1 a
giải được bài toán. Một lần nữa, ta lại xem vế phải và viết  ...  n dưới dạng
a2 a1
2
 a1 an 
  ...  
 a2 a1 
. Vì vậy, ta có thể áp dụng thủ thuật như sau:
a1 an
 ... 
a2 a1
2 2 2
 a1   an   a1 an 
      ...  
a1 a a a a a1 
 ...  2 n   2   ...   1    2 .
a2  a2
2
a1  a1 a  a1 an 
an a1
1   ... 
an
1
a2 a1 a2 a1
a1 an
Bây giờ thì ta chỉ còn lại một bài toán đơn giản: nếu t   ...  , thì
a2 a1
t2 nt
 , hay t  n . Nhưng điều này là hiển nhiên theo BĐT Trung bình cộng,
1 t n 1
trung bình nhân.
Ví dụ 11
Chứng minh rằng với mọi a,b,c>0 ta có bất đẳng thức sau:
 b  c  a
2
(c  a  b ) 2 (a  b  c) 2 3
   .
a 2  (b  c ) 2 b 2  ( a  c ) 2 c 2  ( a  b) 2 5
Nhật Bản, 1997
Lời giải:
Sau khi thất bại vì áp dụng thủ thuật một cách trực tiếp, chúng ta có thể sẽ hơi
thất vọng. Nhưng dạng của bất đẳng thức mời gọi chúng ta. Cách thức tự nhiên
nhất là:
2 2
bc   bc 
(b  c  a) 2 
a
 1   a
 3
 a 2  (b  c)2    b  c 2   
2
  bc 
1   3  
 a   a 
vì trong cách này ta thu được một bất đẳng thức ba biến số mà chúng ta biết khá rõ
bc ca ab
một số tính chất. Như vậy, ta cần chứng minh rằng nếu x  ,y ,z 
a b c
3
, thì  x  y  z  3  ( x  y  z  3) , điều này tương đương với
2 2 2 2

5
  x  15 x  3 xy  18  0 .
2
Đánh tiếc là ta không thể sử dụng trực tiếp dữ kiện xy  yz  zx  12 . Vì vậy, ta cần
tìm các bất đẳng thức dạnh như xy  yz  zx  k ( x  y  z ) . Hằng số tốt nhất là k=2
(vì ta có dấu bằng khi x=y=z=2). Thật thế, sau một số tính toán trực tiếp, bất đẳng
thức có thể viết dưới dạng  a  3abc   ab(a  b) , và đó chính là BĐT Schur.
3

Từ đó, ta có thể viết:


  x   15 x  3 xy  18    x   9 x  18  0 ,
2 2

Và BĐT cuối cùng đúng vì x  y  z  6 .


Tại kỳ thi Toán quốc tế 2001, bài toán 2 đã gây được khá nhiều khó khăn
cho các thí sinh. Lời giải chính thức của HĐGK khá hiệu quả, nhưng rất thiếu tự
nhiên. Ta đưa ra một cách tiếp cận khác, cho phép đưa ra một mở rộng đẹp:
Ví dụ 12
Cho k là số thực, k ≥ 8. Chứng minh rằng với các số nguyên dương a, b, c ta có bất
đẳng thức
a b c 3
  
a  kbc
2
b  kca
2
c  kab
2
k 1
(Mở rộng bài toán IMO 2001 )
Lời giải: Ta có, theo Cauchy-Schwarz



a 

a  kbc 
2

  a a 2  kbc    a  2 

Bây giờ, áp dụng Cauchy-Schwarz cho tổng thứ hai
 a a 2  kbc   
2
a a 3  kabc  2
  a   (a 3
 kabc) 
Như vậy ta chỉ cần chứng min

(k  1)  a   9  (a 3  kabc)
3

Nhưng điều này tương đương với
(k-8)(a3+b3+c3) + 3(k+1)(a+b)(b+c)(c+a)  27kabc
Bất đẳng thức cuối cùng là hiển nhiên teo BĐT trung bình cộng-trung bình nhân.
Tuy nhiên, có lẽ ví dụ khó nhất ở dạng này là bài toán sau:
Ví dụ 13
n n

Cho a1 ,..., an  0 và n>12 sao cho  ak  1 và


k 1
 ka
k 1
k  2 . Chứng minh rằng:

(a2  a1 ) 2  (a3  a2 ) 3  ...  (an  an 1 ) n  0 .


Lời giải:
Ta phải bắt đầu thế nào, nếu như có nhiều điều kiện như vậy? Qui tắc tổng quát
là hãy tạm đừng để ý đến chúng và tập trung vào biểu thức xuất hiện trong yêu cầu
đề bài. Dễ thấy,
(a2  a1 ) 2  (a3  a2 ) 3  ...  (an  an 1 ) n =
 a1 2  a2 ( 2  3)  ...  an 1 ( n  1  n )  an n .
Như thế ta cần chứng minh rằng a1  a2 ( 3  2)  ...  an 1 ( n  n  1)  an n . Để
hoàn thiện vế trái, ta cộng thêm - a1 và an ( n  1  n ) . Như thế, ta cần chứng

a  
n

minh rằng i i  1  i  an n  1  a1 .
i 1

  ai
n n n

Nhưng  ai
i 1
i 1  i  
i 1 i  i 1
. Vì ta có a
k 1
k  1 , rõ ràng ta có thể viết:
2
 n 
n
ai   ai  n
 i 1 
  . Vì vậy, ta cần tìm maximum của a i i và
a  
n
i 1 i  i 1 i  i 1
i 1
i
i 1
n

a
i 1
i i  1 . Nhìn lại điều kiện của bài toán, ta thấy rằng BĐT Cauchy-Schwarz

n n n n
một lần nữa là lựa chọn tốt nhất:  ai i   ai  iai 
i 1 i 1
 ai 
i 1
 ia
i 1
i  2 và
n

tương tự a
i 1
i i  1  3 . Vậy, ta chỉ cần chứng minh rằng 3  2  an n  1  a1 .

2
Nếu chúng ta thử dùng an  , ta sẽ không thành công. Nhưng ta có thể dùng cả
n
hai điều kiện để thấy rằng
n n
1 n 1
2   ai   (i  1)ai  1  (n  1)ai , như thế an  . Từ đó, an n  1  a1  .
i 1 i 1 n 1 n 1
Bây giờ thì ta đã thấy điều kiện n>12 xuất phát từ đâu. Vì n>12 nên ta có
n 1
 3 2.
n 1
Bài tập áp dụng

1. Chứng minh bất đẳng thức:


a b c
( y  z)  ( x  z)  ( x  y )  3( xy  yz  zx) .
bc ca a b
2. Cho x,y,z>0 sao cho x  y  z  x3  y 3  z 3 . Chứng minh rằng:
2 2 2
x z y x z y x yz
   2    2    .
x 1  y  y 1  z  z 1  x 
2
2
x15 xn5
3. Cho n>1, tìm giác trị nhỏ nhất của  ...  , trong đó
x2  ...  xn x1  ...  xn 1
x1 ,..., xn  0 có tổng bình phương bằng 1.
Thổ Nhĩ Kỳ, 1997
1 1 1
4. Cho x,y,z>1 sao cho    2 . Chứng minh rằng
x y z
x  y  z  x 1  y 1  z 1 .
Iran, 1998
5. Cho x, y, z là các số thực dương có tích bằng 1. Chứng minh rằng:
4  9 x 2  4  9 y 2  4  9 z 2  13( x  y  z ) .
6. Cho a,b,c>0 thoả mãn điều kiện abc=1. Chứng minh rằng:
a b c
  1.
1 b  c 1 c  a 1 a  b
7. Cho a,b,c>=0 such that a 2  b 2  c 2  2 max{a 2 , b 2 , c 2 } . Chứng minh rằng:
(a  b  c)(a 2  b 2  c 2 )(a 3  b3  c 3 )  4(a 6  b 6  c 6 ) .
Nhật Bản, 2001
8. Cho n>3 and a1 ,..., an  0 , whose sum of squares is 1. Chứng minh rằng:
a1 a 4
 
2
 ...  2 n  a1 a1  ...  an an .
a 1
2
2 a1  1 5
Đề thi chọn ĐT Rumani, 2002
 1 
9. Chứng minh rằng nếu a1 ,..., a6   , 3  , thì ta có bất đẳng thức sau:
 3 
a1  a2 a2  a3 a3  a4 a4  a5 a5  a6 a6  a1
      0.
a2  a3 a3  a4 a4  a5 a5  a6 a6  a1 a1  a2
10. Chứng minh rằng nếu a, b, c là ba cạnh của tam giác ABC thì
4/3 < a/(a+b) + b/(b+c) + c/(c+a) < 5/3
11. Cho x, y, z là các số thực dương thoả mãn điều kiện
1
i)  z  min{x 2 , y 3} ii ) x  z 3  6 iii) y 3  z 6  20
2
Tìm giá trị lớn nhất của P(x,y,z) = 1/x2+1/y2+1/z2.
VMO 2001
12. Cho x  [0, 1]. Chứng minh rằng
x (13 1  x 2  9 1  x 2 )  16
Olympic 30/4, 1996
13. Chứng minh rằng với 2n số thực a1, a2, …, an, x1, x2, …, xn ta có
a x i i    a   x   n2   a   x 
2
i
2
i i i

Dấu bằng xảy ra khi nào?


Kvant 1989, TH&TT 1997

Các đề toán đề nghị cho cuộc thi giải toán bất đẳng thức
trên Diễn đàn toán học (Hè 2007)

1. Cho x, y là các số thực dương thoả mãn điều kiện x + y = 2. Chứng minh rằng
x15y15(x6 + y6)  2.

2. Cho tam giác ABC có A = 600. I là tâm đường tròn nội tiếp tam giác. Một
đường thẳng thay đổi qua I cắt cạnh AB tại M và AC tại N. Tìm giá trị nhỏ nhất
của biểu thức BM + CN + MN/2.

3. Cho a, b, c, x, y, z là các số thực dương. Chứng minh rằng


a b c
( y  z)  ( z  x)  ( x  y )  3( xy  yz  zx )
bc ca ab

4. Cho a, b, c > 0, abc = 1. Chứng minh rằng


a3 + b3 + c3 + 15  6(1/a + 1/b + 1/c)

5. Cho x1, x2, …, xn; y1, y2, …, yn là các số thực thuộc đoạn [0, 1]. Biết rằng với
mọi r thuộc [0, 1] ta có
| { i I | xi  r} |  | { i I | yi  r} |, trong đó I = {1, 2, ..., n}.
Chứng minh rằng x1 + x2 + ... + xn  y1 + y2 + ... + yn

6. Tìm số thực a > 1 lớn nhất sao cho với mọi số nguyên dương n và với mọi n số
thực x1, x2, …, xn thuộc [1, a] ta có bất đẳng thức
(x1 + x2 + … + xn)(1/x1 + 1/x2 + …+ 1/xn)2  n3

Tóm tắt lời giải.

1. Trước hết ta chứng minh x3y3(x3+y3)  2. Điều này được thực hiện một cách dễ
dàng bằng cách áp dụng bất đẳng thức Cauchy cho các số xy, xy, xy và x 2 – xy +
y2.

Tiếp theo, ta có
x15y15(x6 + y6) = x15y15((x3 + y3)2 – 2x3y3) = x9y9[x3y3(x3+y3)]2- 2x18y18  4x9y9
– 2x18y18 = 2 – 2(x9y9-1)2  2.

Ghi chú. Có thể chứng minh được bất đẳng thức tổng quát:
xn(n-1)/2yn(n-1)/2(xn+yn)  2.

2. Đáp số: min bằng BC đạt được khi IM = IN. Gợi ý: Lấy M’ và N’ đối xứng với
M và N qua IB và IC tương ứng. Sau đó dùng định lý hàm số cos để đánh giá
M’N’.
3. Biến đổi bất đẳng thức về dạng
a b c
(1  )( y  z )  (1  )( z  x )  (1  ( x  y ))  2( x  y  z )  3( xy  yz  zx )
bc ca ab
 yz zx x y
 (a  b  c )     2( x  y  z )  3( xy  yz  zx )
bc ca ab
 yz zx x y
 [(b  c )  (c  a)  (a  b)]     4( x  y  z )  2 3( xy  yz  zx)
bc ca ab
Áp dụng bất đẳng thức Cauchy-Bunhiacovsky-Schwartz, ta có vế trái lớn hơn hay
bằng
 yz  zx x y  2

Như vậy ta cần chứng minh


 yz  zx x y  2
(1)
 4( x  y  z )  2 3( xy  yz  zx)
Đặt m = y + z, n = z + x, p = x + y thì x = (n + p – m )/2, y = (m2 + p2 - n2)/2, z
2 2 2 2 2 2

= (m2+n2-p2)/2. Như vậy,


3(xy + yz + zx) = 3(2m2n2 + 2n2p2 + 2p2n2 – m4 – n4 – p4)/4
Và ta chỉ cần chứng minh rằng
(m  n  p ) 2  2(m 2  n 2  p 2 )  3(2m 2 n 2  2m 2 p 2  2 p 2 n 2  m 4  n 4  p 4 )
Chuyển vế rồi bình phương hai vế (chú ý hai vế đều dương), rút gọn các số hạng
đồng dạng, ta được bất đẳng thức trên tương đương với
m4 + n4 + p4  mnp(m+n+p).
Ghi chú. Bất đẳng thức trên còn có thể chứng minh bằng phương pháp hình học,
cụ thể dùng bất đẳng thức
xa 2  yb 2  zc 2  4 xy  yz  zx .S (1)
Với x, y, z là các số dương, S là diện tích của tam giác có ba cạnh là a, b, c.
(1) có thể được chứng minh như sau: Ta có
xa 2  yb 2  zc 2  ( x  z ) a 2  ( y  z )b 2  2abz cos C  2 ( x  z )( y  z ) .ab  2abz cos C
Như vậy, ta chỉ cần chứng minh
2 ( x  z )( y  z ) .ab  2abz cos C  2 xy  yz  zx ab sin C
 ( x  z )( y  z )  z cos C  xy  yz  zx sin C
Bất đẳng thức cuối cùng đúng theo Cauchy-Bunhiacopsky-Schwartz.

4. Gợi ý, dồn biến hoặc dùng đạo hàm để đưa về bất đẳng thức 1 biến. Lưu ý là bất
đẳng thức 1 biến này khá khoai, cần phải phân trường hợp mới chứng minh được.
Trong đó có kỹ thuật bỏ bớt 1 số số hạng.

5. Giả sử {t1, t2, …, tk} là tập hợp tất cả các giá trị khác nhau mà x i, yi, i = 1, 2, …,
n có thể nhận được, xếp theo thứ tự tăng dần. Đặt
ri = {j  I | xj = ti } và si = {j  I | yj = ti }
Theo giả thiết ta có
rk  sk
rk + rk-1  sk + sk-1
.....
rk + rk-1 + ... + r1 = sk + sk-1 + ... + s1
Cuối cùng, ta có
x1 + x2 + … + xn = r1t1 + r2t2 + … + rktk
= rk(tk – tk-1) + (rk + rk-1)(tk-1-tk-2) + … + (rk + … + r2)(t2-t1) + (rk + … + r2+r1)t1
 sk(tk – tk-1) + (sk + sk-1)(tk-1-tk-2) + … + (sk + … + s2)(t2-t1) + (sk + … + s2+s1)t1
= s1t1 + s2t2 + … + sktk = y1 + y2 + … + yn.

Ghi chú. Bài này thoạt nhìn tưởng hiển nhiên, nhưng trình bày rất khó. Kỹ thuật
trên đây là kỹ thuật tổng Abel

6. Giả sử a đã tìm được. Khi đó chọn x1 = x2 = … = xn-1 = 1, xn = a, ta có


(n-1+a)(n-1+1/a)2  n3
 (a-1)((n2-2n+1)a2 – (2n2-3n+2)a + n-1)  0
 n2(a2-2a) – (2a2-3a-1)n + a2 – 2a – 1  0

Bất đẳng thức này cần đúng với mọi n, do đó ta cần có a  2. Nhiệm vụ bây giờ
cần chứng minh là giá trị a = 2 thoả mãn yêu cầu.

Định lý Euler về tam giác pedal

Trần Nam Dũng


Leonard Euler là một trong các nhà toán học vĩ đại nhất của thế kỷ 18 và của mọi
thời đại. Các công trình của ông bao trùm nhiều lĩnh vực của Toán học, cơ học, vật
lý, trắc địa. Riêng trong toán học, di sản của ông để lại thật đồ sộ với những công
trình trong Giải tích, Lý thuyết số, Đại số, Toán rời rạc và Hình học. Trong các kết
quả của ông có những kết quả rất sâu sắc và cao cấp mà phải học đến chương trình
đại học chuyên ngành toán mới hiểu rõ được (chẳng hạn các công thức nổi tiếng
ei = -1, 1 + 1/4 +1/9 + 1/16 + … = 2/6, M – C + D = 2, định lý Fermat-Euler về
tổng bình phương …) nhưng cũng có những kết quả sơ cấp mà các bạn học sinh
phổ thông cũng có thể hiểu được như đường thẳng Euler, đường tròn 9 điểm Euler,
bài toán về bảy cây cầu ở Konigsberg … Trong những kết quả sơ cấp đó, có một
kết quả rất sâu sắc và đẹp mắt, nhưng ít được biết đến hơn các kết quả trên đây. Đó
là định lý Euler về tam giác pedal. Bài viết này giới thiệu về định lý tuyệt đẹp này
cũng như các kết quả trung gian, qua đó giới thiệu một số định lý nổi tiếng khác
của hình học phẳng như định lý Lagrange, định lý Jacobi, công thức Euler, định lý
Poncelet, định lý Feierbach …

Định lý Euler về tam giác pedal

Cho tam giác ABC và một điểm P bất kỳ trong mặt phẳng tam giác. Hạ MX, MY,
MZ lần lượt vuông góc với BC, CA, AB. Khi đó XYZ được gọi là tam giác pedal
của tam giác ABC ứng với điểm M. Tam giác pedal có nhiều tính chất thú vị,
chẳng hạn

Định lý (Euler). Cho tam giác ABC nội tiếp trong đường tròn tâm O bán kính R.
M là một điểm bất nằm trong mặt phẳng tam giác. Hạ MX, MY, MZ lần lượt
vuông góc với BC, CA, AB. Khi đó diện tích tam giác XYZ có thể tính theo diện
tích tam giác ABC và khoảng cách MO theo công thức sau
1 MO 2
S XYZ  S ABC | 1  |.
4 R2
Chứng minh: Ta ký hiệu SXYZ = S1, SABC = S.

Cách 1: (Tứ giác nội tiếp, tam giác đồng dạng, phương tích, định lý hàm số sin)
Nối dài AM, BM, CM cắt đường tròn ngoại tiếp tại các điểm X’, Y’, Z’ tương ứng.
Ta có ZXM = MBZ (tứ giác BZMX nội tiếp)
MBZ = ABY’(B, Z, A thẳng hàng và B, M, Y’ thẳng hàng)
ABY’ = AX’Y’ (cùng chắn cung AY’)
Từ đó suy ra ZXM = AX’Y’. Tương tự YXM = AX’Z’. Từ đó suy ra
ZXY = Z’X’Y’. Ta sẽ ký hiệu hai góc này là X và X’ tương ứng.
Ta có
S1 = (1/2)XY.XZ.sinX
= (1/2)MB.sinB.MC.sinC.sinX (định lý hàm số sin)
= (1/2)MB.MY’.(MC/MY’).sinB.sinC.sinX
= (1/2) |MO2 – R2| (BC/Z’Y’).sinB.sinC.sinX (phương tích, MBC ~ Z’MY’)
= (1/2) |MO2 – R2|.BC.sinC.sinB.(sinX’/Y’Z’)
= (1/8) |1 – MO2/R2| AC.BC.sinC (sinB = AC/2R, sinX’/Y’Z’ = 1/2R)
= (1/4)S |1 – MO2/R2|.

Cách 2: (Véc-tơ, tâm tỷ cự)


Gọi X’, Y’, Z’ là các điểm đối xứng của M qua X, Y, Z tương ứng. Khi đó S 1 =
SX’Y’Z’/4. Nhưng diện tích tam giác X’Y’Z’ bằng diện tích lục giác AY’CX’BZ’ trừ
đi diện tích các tam giác AY’Z’, BZ’X’, CX’Y’, diện tích lục giác AY’CX’BZ’
bằng 2 lần diện tích tam giác ABC, tam giác AY’Z’ là tam giác cân có
AY’=AZ’=MA và góc Y’AZ’ = 2A, nên ta có
1 1 1 1 1
S1  S X 'Y 'Z '  (2 S  MA 2 sin 2 A  MB 2 sin 2 B  MC 2 sin 2C )
4 4 2 2 2
Vì OA sin 2 A  OB sin 2 B  OC sin 2C  0 nên theo định lý Lagrange về tâm tỉ cự ta

MA 2 sin 2 A  MB 2 sin 2 B  MC 2 sin 2C
 (sin 2 A  sin 2 B  sin 2C ) MO 2  OA 2 sin 2 A  OB 2 sin 2 B  OC 2 sin 2C
 ( MO 2  R 2 )4 sin A sin B sin C
4abc
 ( MO 2  R 2 )
8R 3
 MO 2 
 1  2 S
 R 2 
Suy ra
1  MO 2  1  MO 2 
S1   2 S  S 1  2   S 1  2  .
4  R  4  R 
Cách 3: (Phương pháp toạ độ)
Chọn hệ trục toạ độ nhận đường cao AH làm trục tung và BC làm trục hoành. Khi
đó ta có thể đặt toạ độ các đỉnh A, B, C lần lượt là: A (0, a), B (b, 0), C (c, 0).
Điểm có toạ độ M (m, n).
Khi đó dễ dàng tính được: S = (1/2)| a(b – c)|,
2 2
b  c bc  a 2 bc  a 2  bc  (a 2  b 2 )(a 2  c 2 )
O( , ), R2       
2 2a  2   2a  4a 2
Tiếp theo ta tính toạ độ các hình chiếu X, Y, Z. Dễ thấy X (m, 0). Đường thẳng AC
có pháp véc tơ là (c, - a). MY vuông góc với AC suy ra MY  t (a, c ) , suy ra
Y(m+at, n+ct). Vì Y thuộc đường thẳng AC có phương trình a(x – 0) + c(y – a) = 0
nên
a(m+at) + c(n+ct – a) = 0
suy ra
t = (ac – am – cn)/(a2+c2)
Từ đó ta tính được
 a(ac  am  cn a(c 2  an  cn 
XY   , 
 a2  c2 a2  c2 
Tương tự
 a (ab  am  bn a (b 2  an  bn 
XZ   , 
 a2  b2 a2  b2 
Sử dụng công thức tính diện tích tam giác SABC = (1/2)| x1y2 – x2y1| với
AB  ( x1 , y1 ), AC  ( x 2 , y 2 )
ta tính được
1 a2
S1  | acb 2  a 2 cn  abcm  ab 2 m  a 2 mn  abm 2  cb 2 m  can 2 
2 (a  b )(a  c )
2 2 2 2

cbmn  abc 2  a 2 bn  abcm  ac 2 m  a 2 mn  acm 2  bc 2 m  ban 2  bcmn |


1 a2
 | a(b  c) || bc  m 2  n 2  (b  c)m  (a  bc / a)n |
2 (a  b )(a  c )
2 2 2 2

1 a2 bc 2 a 2  bc 2 bc 2 a 2  bc 2
 | a (b  c ) || ( m  )  ( n  )  ( )  ( ) |
2 (a 2  b 2 )(a 2  c 2 ) 2 2a 2 2a
2S 1 OM 2
 | MO 2
 R 2
| S | 1  |.
8R 2 4 R2
Định lý Euler là một kết quả thú vị và sâu sắc của hình học trong tam giác. Định lý
này có nhiều hệ quả hay. Chúng ta sẽ xem xét một số hệ quả đó:

Đường thẳng Simson

Từ kết quả của định lý Euler, ta thấy nếu M nằm trên đường tròn ngoại tiếp tam
giác ABC, tức là nếu OM = R thì diện tích tam giác pedal bằng 0. Điều đó có
nghĩa là tam giác XYZ suy biến thành đường thẳng. Và như vậy, ta đã chứng minh
được một kết quả quen thuộc sau đây:

Định lý (đường thẳng Simson). Cho tam giác ABC nội tiếp trong đường tròn (C).
M là một điểm bất kỳ trên C. Hạ MX, MY, MZ lần lượt vuông góc với BC, CA,
AB. Khi đó X, Y, Z cùng nằm trên một đường thẳng.

Đường thẳng đi qua X, Y, Z được gọi là đường thẳng Simson ứng với điểm M.
Tính chất thú vị này có thể chứng minh khá dễ dàng mà không cần thông qua định
lý Euler, sử dụng tính chất của tứ giác nội tiếp.

Đường thẳng Simson có nhiều tính chất đẹp, trong đó hai tính chất sau đây:
1) Nếu gọi H là trực tâm tam giác ABC thì đường thẳng Euler của điểm M
chia đôi đoạn thẳng HM.
2) Khi điểm M di động trên đường tròn ngoại tiếp vẽ một cung có độ lớn  thì
đường thẳng Simson ứng với M quay một góc /2. Một hệ quả thú vị của
tính chất này là “đường thẳng Simson của hai điểm xuyên tâm đối thì
vuông góc với nhau”.

Công thức Euler và định lý Poncelet

Khi M trùng với I là tâm đường tròn nội tiếp tam giác thì XYZ là tam giác nội tiếp
trong đường tròn tâm I bán kính r, có các góc X, Y, Z tương ứng bằng 90 0 – A/2,
900 – B/2, 900 – C/2 nên ta có
SXYZ = 2r2sinXsinYsinZ = 2r2cos(A/2)cos(B/2)cos(C/2) = S.r/2R.
Từ đó, thay vào định lý Euler, ta được
Sr/2R = (1/4)S(1 – IO2/R2)
 2Rr = R2 – IO2
 IO2 = R2 – 2Rr
Ta thu được một kết quả đẹp mắt khác của hình học phẳng (được gọi là công thức
Euler)

Định lý. (Công thức Euler) Cho tam giác ABC nội tiếp trong đường tròn tâm O
bán kính R và ngoại tiếp đường tròn tâm I bán kính r. Khi đó khoảng cách IO có
thể tính theo công thức sau:
IO2 = R2 – 2Rr.

Công thức Euler, ngoài cách chứng minh thông qua định lý Euler về tam giác
pedal trên đây, còn có thể chứng minh bằng cách sử dụng tâm tỷ cự hoặc phương
tích. Dưới đây chúng ta trình bày ngắn gọn 2 chứng minh này:

Cách 1 – Dùng tâm tỷ cự. Ta biết rằng


a IA  b IB  c IC  0
Áp dụng định lý Jacobi về tâm tỷ cự, ta có
aOA2 + bOB2 + cOC2 = (a+b+c)IO2 + (abc2+bca2+cab2)/(a+b+c)
Suy ra
IO2 = R2 – abc/(a+b+c) = R2 – 4RS/(2S/r) = R2 – 2Rr.

Cách 2. – Dùng phương tích và tam giác đồng dạng.


Nối dài AI cắt đường tròn (O) tại điểm thứ hai A 1, nối dài A1O cắt (O) tại điểm thứ
hai A2. Giả sử đường tròn (I) tiếp xúc với AB tại J. Khi đó tam giác AJI đồng dạng
với tam giác A2BA1 (do A = A1 và J = B). Vì thế ta có
IA/IJ = A1A2/A1B
Suy ra
IA.A1B = 2Rr
Mặt khác IBA1 = A/2 + B/2 = BIA1 suy ra tam giác IA1B cân tại A1, suy ra A1B
= IA1,
Như thế ta có
IA.IA1 = 2Rr
Nhưng IA.IA1 là phương tích của điểm I đối với đường tròn (O) và bằng R 2 – IO2
và từ đây ta có điều phải chứng minh.

Từ công thức Euler, ta thấy rằng khoảng cách giữa tâm đường tròn nội tiếp và tâm
đường tròn ngoại tiếp không phụ thuộc trực tiếp vào độ dài các cạnh của tam giác
mà chỉ phụ thuộc vào R và r. Từ đó, có thể suy luận ra rằng sẽ có vô số tam giác
nội tiếp trong các đường tròn (O, R), (I, r) với IO 2 = R2 – 2Rr. Và quả thật ta có
định lý đẹp mắt sau:

Định lý. (Poncelet) Cho tam giác ABC nội tiếp trong đường tròn tâm O bán kính R
và ngoại tiếp đường tròn tâm I bán kính r. Khi đó tồn tại vô số các tam giác A’B’C’
nội tiếp trong (O, R) và ngoại tiếp (I, r). Hơn nữa, một điểm bất kỳ trên (O) là đỉnh
của một tam giác như vậy.

Định lý này có thể phát biểu tổng quát cho một đa giác bất kỳ và được gọi là định
lý lớn Poncelet. Tuy nhiên, ở đây, ta chỉ đề cập đến phép chứng minh định lý
Poncelet cho tam giác, liên quan trực tiếp đến công thức Euler.

Chứng minh: Giả sử ta có điều kiện của định lý. Khi đó IO 2 = R2 – 2Rr. Lấy một
điểm A’ bất kỳ trên (O). Kẻ các tiếp tuyến A’J, A’K đến (I) cắt (O) tại các điểm thứ
hai B’, C’. Ta sẽ chứng minh IC là phân giác của góc A’C’B’, từ đó suy ra C’B’
tiếp xúc (I).

Nối dài A’I cắt (O) tại A1, nối dài A1O cắt (O) tại A2. Ta có
IA’.IA1 = R2 – IO2 = R2 – (R2 – 2Rr) = 2Rr.
Hai tam giác A’JI và A2C’A1 đồng dạng với nhau, suy ra
A’I/IJ = A2A1/A1C’
 A’I.A1C’ = 2Rr
Từ đó ta suy ra IA1 = A1C, suy ra C’IA1 = IC’A1. Nhưng
C’IA1 = IA’C’ + A’C’I (góc ngoài của tam giác)
IC’A1 = IC’B’ + B’C’A1
IA’C’ = B’C’A1 (góc chắn các cung A1C’ và A1B’ bằng nhau)
Nên từ đây ta có A’C’I = IC’B’, tức là IC’ là phân giác góc A’C’B’ (đpcm).

Định lý Feierbach

Sử dụng công thức Euler và một số công thức khác, ta có thể chứng minh được
một kết quả tuyệt vời khác của hình học tam giác, đó là định lý Feierbach. Trong
tam giác có 6 đường tròn đặc biệt là đường tròn ngoại tiếp, đường tròn nội tiếp, 3
đường tròn bàng tiếp và đường tròn 9 điểm Euler. Đường tròn 9 điểm Euler thu
được từ đường tròn ngoại tiếp bằng một phép vị tự tâm H (trực tâm tam giác ABC)
tỷ số 1/2. Nhưng còn một tính chất rất bất ngờ khác của đường tròn Euler là:
Định lý. (Feierbach) Đường tròn 9 điểm Euler tiếp xúc với đường tròn nội tiếp và
cả 3 đường tròn bàng tiếp.

Chứng minh. Ta sử dụng công thức Euler và các công thức tương tự (chứng minh
bằng cách sử dụng tâm tỷ cự)
IO2 = R2 – 2Rr
OG2 = R2 – (a2 + b2 + c2)/9
IG2 = (p2 + 5r2 – 16Rr)/9
Ta biết rằng tâm E của đường tròn Euler là trung điểm của HO, còn trọng tâm G
thì chia đoạn HO theo tỷ số 2:1. Từ đó suy ra 3EG  EO  0 . Suy ra E là tâm tỷ
cự của {G, O} với bộ trọng lượng tương ứng là (3, -1). Áp dụng định lý Lagrange
về tâm tỷ cự, ta có
3IG2 – IO2 = (3-1)IE2 + 3EG2 – EO2
Suy ra
2IE2 = (p2 +5r2 – 16Rr)/3 – (R2 – 2Rr) + (9R2 – a2 – b2 – c2)/6
= R2/2 + 2r2 – 2Rr + (p2 – r2 – 4Rr)/3 – (a2+b2+c2)/6
Nhưng p2 – r2 – 4Rr = p2 – S2/p2 – abc/p = p2 – (p-a)(p-b)(p-c)/p – abc/p = [p3 – (p3
– p2(a+b+c) + p(ab+bc+ca) – abc) – abc]/p = p(a+b+c) – (ab+bc+ca) =
(a2+b2+c2)/2
Nên ta có
2IE2 = R2/2 + 2r2 – 2Rr
Suy ra IE2 = R2/4 + r2 – Rr = (R/2 – r)2 và từ đó IE = R/2 – r. Đẳng thức này chứng
tỏ đường tròn nội tiếp (I, r) tiếp xúc trong với đường tròn Euler (E, R/2). Kết quả
với các đường tròn bàng tiếp được chứng minh hoàn toàn tương tự.

Như vậy, từ định lý Euler về tam giác pedal, chúng ta đã đi qua những kết quả
tuyệt đẹp trong hình học tam giác như đường thẳng Simson, công thức Euler, định
lý Poncelet, định lý Feierbach. Xuyên suốt các phép chứng minh ta thấy có ứng
dụng của phương tích, tam giác đồng dạng, tứ giác nội tiếp, tam giác lượng và đặc
biệt là các định lý về tâm tỉ cự. Xin nhắc lại hai định lý này:

Định lý Lagrange. Giả sử G là tâm tỉ cự của hệ điểm A 1, A2, …, An với các trọng
lượng m1, m2, …, mn tương ứng. Khi đó với mọi điểm M ta có
n
 n  n

i 1
m i MAi
2
   m
 i 1 
i  MG 2
 
i 1
mi GAi2

Định lý Jacobi. Giả sử G là tâm tỉ cự của hệ điểm A 1, A2, …, An với các trọng
lượng m1, m2, …, mn tương ứng. Khi đó với mọi điểm M ta có
n
 n  1 n

 m i MAi
2
   m
 i 1 
i  MG 2

 n 
 i , j 1
mi m j Ai A 2j
  mi 
i 1

 i 1 
Cuối cùng, xin giới thiệu một số kết quả thú vị khác liên quan đến các vấn đề đã
được đề cập trong bài viết này dưới dạng bài tập.

Bài tập

1. Cho tam giác ABC, M là một điểm bất kỳ trong mặt phẳng tam giác, XYZ là
tam giác pedal của tam giác ABC ứng với điểm M. Các đường đối phân giác của
AM, BM, CM đồng quy tại một điểm M1, X1Y1Z1 là tam giác pedal của tam giác
ABC ứng với điểm M1. Chứng minh rằng 6 điểm X, Y, Z, X 1, Y1, Z1 cùng nằm trên
1 đường tròn (gọi là đường tròn pedal ứng với điểm M cũng như ứng với điểm
M1).

2. Cho 4 điểm trong mặt phẳng, trong đó không có 3 điểm nào thẳng hàng. Chứng
minh rằng các đường tròn pedal của 1 điểm tuỳ ý trong chúng ứng với tam giác tạo
bởi 3 đỉnh còn lại đồng quy tại một điểm.

3. Cho tam giác ABC. M là một điểm bất kỳ trong mặt phẳng tam giác. Chứng
minh rằng
S ( MBC ).MA  S ( MCA).MB  S ( MAB ).MC  0 .

4. Cho tam giác ABC nội tiếp đường tròn (O) và ngoại tiếp đường tròn (I). Chứng
minh rằng tồn tại vô số các tam giác A’B’C’ cũng nội tiếp đường tròn (O) và ngoại
tiếp đường tròn (I). Hơn nữa, chứng minh rằng trọng tâm của các tam giác này
cùng nằm trên một đường tròn.

5. Cho tam giác ABC nội tiếp trong đường tròn (O). Chứng minh rằng trên (O) tồn
tại đúng 3 điểm sao cho các đường thẳng Simson tương ứng với chúng tiếp xúc
với đường tròn 9 điểm Euler. Hơn nữa, chứng minh rằng 3 điểm này tạo thành 3
đỉnh của một tam giác đều.

Vai trò của các bài toán tổ hợp trong việc


rèn luyện tư duy toán học và kỹ năng giải toán

Trần Nam Dũng

Mở đầu

Các bài toán tổ hợp từ lâu đóng một vai trò quan trọng trong việc rèn luyện tư duy
toán học và kỹ năng giải toán. Bài toán tổ hợp có một số đặc điểm quan trọng
mang tính khác biệt sau:
+ Không đòi hỏi nhiều kiến thức, do đó có thể giảng dạy tại các bậc lớp khác
nhau.
+ Không có những khuôn mẫu nhất định cho việc giải (giống như việc giải
phương trình, khảo sát hàm số, tính tích phân), do vậy luôn đòi hỏi sự sáng tạo từ
phía học sinh.
+ Thường được phát biểu bằng lời văn, đòi hỏi học sinh phải có kỹ năng đọc,
hiểu và rút trích thông tin, biết cách phát biểu lại bằng ngôn ngữ toán học. Bài toán
tổ hợp thường mang tính thực tế và tính thẩm mỹ cao, khiến học sinh yêu thích,
ghi nhớ.

Vì vậy, tại các kỳ thi Olympic Toán ở các nước, các bài toán tổ hợp luôn xuất hiện
với một tỷ lệ khá cao. Tuy nhiên, ở Việt Nam, các bài toán tổ hợp xuất hiện khá ít.
Điều này có thể thấy rõ thông qua việc nghiên cứu các đề thi học sinh giỏi các tỉnh
thành, đề thi học sinh giỏi quốc gia, các đề toán trên báo Toán học và Tuổi trẻ.
Theo sự góp ý của nhiều đồng nghiệp nước ngoài, đề thi Olympic Toán của Việt
Nam mang nặng tính kỹ thuật, rất ít màu sắc thực tế và vì vậy cũng thiếu luôn cả
vẻ đẹp toán học. Đây là điều chúng ta cần bàn vì Toán học không chỉ là các bài
toán khô khan, mà là cuộc sống, thực tế và vẻ đẹp.

Toán tổ hợp hoàn toàn không được dạy ở các lớp thường (với mục tiêu luyện thi
đại học, mà ở đó, tổ hợp nghĩa là các hệ số C, A, P … mà thôi), được dạy với một
tỷ lệ đối phó ở các lớp chuyên. Theo tôi, đó một sai lầm trong chương trình đào
tạo. Và sai lầm này bắt nguồn từ cách ra đề thi và cách dạy “hướng đến thi cử”
hiện nay của chúng ta.

Theo tôi, học sinh phải được làm quen với toán tổ hợp từ các lớp cấp 2. Những
nguyên lý Dirichlet, nguyên lý cực hạn, phương pháp tô màu, bất biến … hoàn
toàn có thể đưa vào một cách tự nhiên cho các bạn học sinh THCS. Với các lớp
thường, từ lớp 10 cần phải dạy kỹ các chủ đề: Đại số mệnh đề, các phương pháp
chứng minh, Tập hợp, Ánh xạ, Phép đếm, Xác suất cổ điển và Thống kê mô tả.
Những chủ đề rời rạc này rất quan trọng cho việc hình thành tư duy toán học và kỹ
năng trình bày vấn đề cho học sinh. Cụ thể học sinh phải biết các phương pháp
chứng minh và suy luận cơ bản sau đây

+ Phép chứng minh phản chứng


+ Phép chứng minh dùng mệnh đề phản đảo
+ Quy nạp toán học
+ Tổng quát hoá và đặc biệt hoá
+ Suy luận backward

Theo kinh nghiệm của cá nhân tôi và các đồng nghiệp, việc nắm vững các chủ đề
nêu trên giúp học sinh có thể hiểu rõ các vấn đề ở bậc học phổ thông (như hàm số,
giải tích tổ hợp …) và bậc đại học sau này.
Các bài toán tổ hợp ngoài việc phát triển các kỹ năng như nói trên, còn mang tính
thực tế và tính thẩm mỹ cao (rất thú vị khi đọc đề bài), vì thế đem lại cho các học
sinh niềm đam mê, sự hứng thú. Tôi tin chắc rằng một bài toán về xác suất sẽ luôn
dễ nhớ và luôn đem đến những cuộc tranh luận hấp dẫn hơn bất cứ loại toán nào
khác.

Phân loại các bài toán tổ hợp

Các bài toán tổ hợp rất đa dạng, có thể chia thành các nhóm chủ đề sau
+ Các bài toán đếm, các bài toán về tập hợp, ánh xạ
+ Các bài toán về đồ thị, đường đi, quan hệ
+ Các bài toán trò chơi
+ Các bài toán tô màu
+ Các bài toán về phủ hình
+ Các bài toán hình học tổ hợp (góc, độ dài cạnh, đường kính, diện tích …)
+ Các bài toán trên bảng vuông
+ Các bài toán xác suất
+ Các bài toán trên lưới nguyên
+ Các bài toán liên quan đến định lý Pick, Helly, Ramsey …

Các phương pháp giải bài toán tổ hợp

+ Các phương pháp đếm


+ Các phương pháp đồ thị
+ Chiến thuật “backward”
+ Nguyên lý bất biến
+ Phương pháp tô màu
+ Nguyên lý cực hạn
+ Nguyên lý Dirichlet
+ Quy nạp toán học
Bài tập minh hoạ

1. Một nhà ảo thuật cùng với trợ lý của anh ta muốn thực hiện một trò ảo thuật như
sau. Một khán giả viết lên bảng một số có N chữ số. Người trợ lý che hai chữ số
liền nhau bằng các hình tròn màu đen. Sau đó nhà ảo thuật xuất hiện. Nhiệm vụ
của anh ta là đoán hai chữ số bị che (và cả thứ tự của chúng). Với giá trị nhỏ nhất
nào của N nhà ảo thuật có thể, với sự thông đồng với trợ lý, luôn thực hiện được
màn ảo thuật thành công?

2. Hai người chơi một trò chơi sau: Người thứ nhất nghĩ ra một số nguyên nằm
trong khoảng từ 1 đến 144. Người thứ hai có thể hỏi câu hỏi dạng Có/không bằng
cách sau: Chọn ra một tập con M bất kỳ của {1, 2, …, 144} và hỏi: Con số của bạn
nghĩ có nằm trong M không? Người thứ nhất sẽ trả lời đúng theo thực tế. Nếu câu
trả lời là đúng, người thứ hai phải trả 2.000 đồng, nếu câu trả lời là sai, người thứ
hai phải trả 1.000. Hỏi người thứ hai phải đặt các câu hỏi thế nào để chắc chắn tìm
được số mà người thứ nhất nghĩ với số tiền phải trả ít nhất.

3. (Trường hợp phẳng của bài số 6, IMO 2007). Cho tập hợp
M = {(x, y)| 0  x, y  n, x2+y2  0}
Hỏi cần ít nhất bao nhiêu đường thẳng để phủ hết các điểm của M nhưng không
phủ gốc toạ độ?

4. Trên bàn có 2001 đồng tiền. Hai người chơi trò chơi sau. Hai người thay phiên
nhau đi. Mỗi lần đi, người thứ nhất có thể lấy đi một số lẻ bất kỳ các đồng tiền từ 1
đến 99, người thứ hai – một số chẵn bất kỳ từ 2 đến 100. Người nào không đi được
nữa thì thua. Hỏi người nào thắng nếu chơi đúng?
Hướng dẫn: Người thứ nhất thắng: Chiến thuật – Đầu tiên lấy 81 viên, sau đó lấy
101 – x viên, trong đó x là số viên mà người thứ hai lấy ở lượt trước.

5. Trên bàn có 100 viên kẹo. Hai người cùng thay phiên nhau bốc đi k viên kẹo,
trong đó k  {1, 2, 3}. Hỏi ai là người có chiến thuật thắng? Cùng câu hỏi trên với
k  {1, 2, a}, là số nguyên dương cho trước.

6. Tìm số tất cả các dãy {x1, x2, …, xn} với xi thuộc {a, b, c}, i = 1, 2, …, n thoả
mãn điều kiện x1 = xn = a và xi  xi+1 với i = 1, 2, …, n-1.
Giải: Gọn un là số các dãy thoả mãn điều kiện, vn là số các dãy bắt đầu bằng a, tận
cùng bằng b hoặc c và thoả mãn điều kiện trên. Khi đó rõ ràng
1) un = vn-1 (chỉ cần thêm a vào phía sau)
2) vn = 2un-1 (thêm b hoặc c vào xâu dạng u) + vn-1 (thêm b nếu xâu v tận
cùng bằng c và thêm c nếu xâu v tận cùng bằng b). Từ đó ta tìm được công thức
truy hồi un+1 = un + 2vn-1. Có thể có nhiều cách lý luận khác.

7. Số nào lớn hơn? Số các tam giác với cạnh nguyên có chu vi 1997 hay có chu vi
2000?
Đáp số: Bằng nhau – xây dựng 2 đơn án từ 1997  2000 và ngược lại.

8. Có bao nhiêu cách chọn ra 3 người từ 10 xếp thành một hàng dọc sao cho không
có 2 người đứng cạnh nhau trên hàng được chọn.

9. Phương trình x1 + x2 + … + xn = k có bao nhiêu nghiệm nguyên không âm?

10. Hãy tính  | A  B | . Cùng câu hỏi với |A| = p, |B| = q.


A , B {1, 2 ,..., n}
11. Cho bảng vuông 5 x 5. Hỏi có thể tô màu 16 ô của bảng sao cho trong mỗi
bảng 2 x 2 có nhiều nhất 2 ô được tô?

Một số đề thi Olympic của các nước năm 2007 (phần tổ hợp)

1. (Estonia 2007) Xét lưới vuông 10 x 10. Với mỗi nước đi, ta tô màu 4 hình
vuông đơn vị nằm ở giao của 2 hàng và 2 cột. Một nước đi là hợp lệ nếu ít nhất 1
trong 4 hình vuông này trước đó không được tô. Hỏi số nước đi lớn nhất có thể để
tô màu toàn bộ lưới vuông bằng bao nhiêu?

2. (Thái Lan 2006) 229 học sinh nam và 271 học sinh nữ được chia thành 10
nhóm, mỗi nhóm 50 học sinh được đánh số từ 1 đến 50. Người ta muốn chọn ra
một nhóm 4 học sinh, trong đó số học sinh nữ được chọn là lẻ và thoả mãn điều
kiện sau đây: 4 người này được chọn từ 2 nhóm và có 2 cặp học sinh có cùng số
thứ tự. Chứng minh rằng số cách chọn các nhóm như vậy là một số lẻ.

3. (Moldova 2007) Chứng minh rằng hình vuông cạnh 14 có thể phủ được bởi 21
hình vuông: 6 hình vuông cạnh 1, 5 hình vuông cạnh 2, …, 1 hình vuông cạnh 6.

4. (UK 2007) Ở nước Hexagonia, 6 thành phố được kết nối bởi hệ thống đường sắt
sao cho giữa hai thành phố bất kỳ có đường sắt nối trực tiếp. Vào ngày chủ nhật,
một số đoạn đường đóng cửa để sửa chữa. Luật đường sắt quy định là mọi thành
phố phải có thể đến được từ một thành phố khác (không nhất thiết trực tiếp) trong
mọi thời điểm. Có bao nhiêu cách đóng một vài đoạn đường để yêu cầu này được
thoả mãn?

5. (Nhật Bản 2007) Có bao nhiêu cách biểu diễn 100 dưới dạng tổng của các luỹ
thừa tự nhiên của 3 (Ta coi hai cách biểu diễn là như nhau nếu chúng chỉ khác thứ
tự)
6. (Nhật Bản 2007) Có ba hình chữ nhật trên mặt phẳng có cạnh song song với
trục Ox hoặc trục Oy. Ba hình chữ nhật này có thể chia mặt phẳng này thành nhiều
nhất bao nhiêu phần?

7. (Thổ Nhĩ Kỳ 2007) Xác định số cách điền các số 1 và -1 vào các hình vuông
đơn vị của bàn cờ 2007x2007 sao cho trị tuyệt đối của tổng các số trong mọi hình
vuông tạo thành từ các hình vuông đơn vị của bàn cờ không vượt quá 1.

8. (Hàn Quốc 2007) Cho hình vuông 4 x 4. Có bao nhiêu cách điền các số 0 và 1
vào các ô sao cho tích của hai số ở hai ô kề nhau (có cạnh chung) luôn bằng 0?

9. (Áo 2007) Cho một n giác lồi với một tam giác phân, nghĩa là một cách chia n
giác này ra thành các tam giác bằng các đường chéo không giao nhau. Chứng
minh rằng n đỉnh của n giác lồi có thể được đánh số bởi các chữ số của 2007 sao
cho mọi tứ giác tạo thành từ hai tam giác của tam giác phân và có cạnh chung có
các đỉnh được đánh số bởi các chữ số có tổng bằng 9.

10. (Iran 2007) Có n đường thẳng trên mặt phẳng, trong đó không có 2 đường
thẳng nào song song và 3 đường thẳng nào đồng quy. Với mỗi hai đường thẳng,
gọi góc giữa chúng là góc nhỏ nhất tại giao điểm của chúng. Hãy tìm giá trị lớn
nhất của tổng tất cả Cn2 góc giữa các đường thẳng.

11. (Iran 2007) Cho I1, I2, …, In là n đoạn thẳng đóng trên R sao cho từ bất kỳ k
đoạn thẳng trong số chúng, luôn tìm được 2 đoạn có điểm chung. Chứng minh
rằng có thể tìm được k – 1 điểm trên R sao cho mỗi đoạn thẳng đã cho chứa ít nhất
một điểm trong các điểm được chọn.

12. (Thuỵ Sĩ 2006) Cho số nguyên dương n. Tìm số tất cả các tập con A  {1, 2,
…, 2n} sao cho không tồn tại x, y  A với x + y = 2n+1.

13. (Thuỵ Sĩ 2006) 3n điểm trên đường tròn chu vi 6n chia đường tròn thành n
cung độ dài 1, n cung độ dài 2 và n cung độ dài 3. Chứng minh rằng trong số các
điểm này có hai điểm nằm ở vị trí xuyên tâm đối.

14. (Bugaria 2007) Tìm số cạnh nhỏ nhất có thể của đồ thị với n đỉnh thoả mãn
điều kiện sau:
a) Nếu ta vẽ thêm một cạnh bất kỳ thì sẽ có thêm một tam giác mới (3-
clique).
b) Nếu ta vẽ thêm một cạnh bất kỳ thì sẽ có thêm một 4-clique.

15. (Bulgaria 2007) Một đường tròn được gọi là được tô tốt nếu các đỉnh của mọi
tam giác đều nội tiếp trong đường tròn được tô bởi các màu khác nhau. Cho k là
đường tròn có bán kính bằng 2.
a) Tồn tại hay không một cách tô các điểm của k và nằm trong k bằng 3
màu sao cho k và mọi đường tròn có bán kính ít nhất 1 và tiếp xúc với k là được tô
tốt?
b) Cùng câu hỏi với 7 màu.

16. (Bulgaria 2007) Tìm số nguyên dương m nhỏ nhất sao cho mọi 5 tam giác đều
có tổng diện tích m có thể phủ được tam giác đều có diện tích 1.

17. (CH Séc 2007) Với những giá trị nào của n ta có thể phân hoạch tập hợp M =
{1, 2, …, n} thành a) hai b) ba tập con rời nhau có cùng số phần tử và mỗi một
trong chúng chứa trung bình cộng của các phần tử của chúng.

Tài liệu tham khảo


1. G.Polya, Giải bài toán như thế nào?, Nhà xuất bản giáo dục 1997.
2. G.Polya, Toán học và những suy luận có lý, Nhà xuất bản giáo dục 1995
3. G.Polya, Sáng tạo Toán học, Nhà xuất bản giáo dục 1997
4. A.M. Iaglom and I.M.Iaglom, Challenging Mathemtical problems with
elementary solutions, Volume I: Combinatorial Analysis and Probability Theory,
Dover Publications, 1987.
5. Arthur Engel, Problem-Solving Strategies, Springer 1997
6. B.Averbach and O.Chein, Problem Solving Through Recreational Mathematics,
Dover Publications 2000.
7. N.Agakhanov và tập thể tác giả, Olympic Toán toàn Nga 1993-2006, Nhà xuất
bản MCCME 2007.
8. I.Voronovic và tập thể tác giả, Các bài toán thi Olympic toán thành phố Minsk
2006, Minsk 2006.
9. K.Kokhas và tập thể tác giả, Các bài toán thi Olympic toán thành phố Saint
Peterburg 2003, Saint-Peterburg 2003.
10. V.N.Sachkov, Nhập môn các phương pháp tổ hợp của toán rời rạc, Nhà xuất
bản MCCME 2004.
11. Các bài toán thi Olympic Toán các nước và Olympic Toán quốc tế năm 2006,
2007.
12. Tạp chí Toán học và Tuổi trẻ, Kvant, Komal, Crux.
13. Tài liệu Internet, đặc biệt là các websites: www.diendantoanhoc.net,,
www.mathlinks.ro, www.animath.fr, mathcircle.berkeley.edu, www.cms.math.ca.

The 17th Japanese Mathematical Olympiad

Vòng 1 (8/1/2007)

Thời gian làm bài: 3 giờ


Mỗi một bài giá trị 1 điểm. Không cần phải ghi lời giải (chỉ cần đáp số).

Đề bài

1. Cho ABCD là tứ giác lồi với AB = 3, BC = 4, CD = 5, DA = 6 và ABC = 900.


Tìm diện tích tứ giác ABCD.

2. Xác định chữ số hàng chục của số 1112 (luỹ thừa bậc 1213 của 11, không phải
13

là luỹ thừa 13 của 1112.)

3. AB là một đoạn thẳng độ dài 7 trên mặt phẳng, và P là điểm sao cho khoảng
cách giữa P và đường thẳng AB bằng 3. Hãy tìm giá trị nhỏ nhất của AP x PB.
4. n là một số có 4 chữ số, trong đó chữ số hàng chục khác 0, và nếu ta lấy 2 chữ
số đầu tiên và 2 chữ số cuối cùng như hai số có 2 chữ số thì tích của chúng là ước
số của n. Tìm tất cả n với tính chất như vậy.

5. 3 hình chữ nhật được vẽ trên mặt phẳng, và 2 hình chữ nhật bất lỳ có cạnh song
song với nhau. Chúng chia mặt phẳng ra thành một số phần. Tìm số phần mặt
phẳng lớn nhất có thể. (Chúng ta coi phần mặt phẳng không nằm trong bất cứ một
hình chữ nhật nào cũng là một phần. Ví dụ, nếu ta chỉ có 1 hình chữ nhật thì mặt
phẳng được chia thành 2 phần).

6. Ta có 15 tấm thẻ được đánh số 1, 2, …, 15. Có bao nhiêu cách chọn ra một số (ít
nhất 1) tấm thẻ sao cho tất cả các số viết trên các tấm thẻ này đều lớn hơn hoặc
bằng số tấm thẻ được chọn.

7. Có bao nhiêu cách biểu diễn số 100 dưới dạng tổng các luỹ thừa không âm của
3? (Hai cách biểu diễn chỉ khác nhau thứ tự được coi là một.)

8. Có bao nhiêu cách để cắt hình lập phương S thành các tứ diện {T 1, T2, …, Tk}
với các tính chất sau?
(1) Mọi đỉnh của T1, …, Tk là một đỉnh của S
(2) Với mọi i  j, giao của Ti và Tj là một mặt chung của chúng, một cạnh
chung của chúng, một đỉnh chung hoặc là tập rỗng.

9. Có bao nhiêu cặp số nguyên (a, b) thoả mãn điều kiện a2b2 = 4a5 + b3.

10. Có một số tấm thẻ, trên đó có ghi các số nguyên dương. Tổng các số nguyên
dương đó bằng 2007. Với mỗi số nguyên 1  k  2006 ta có thể chọn một số các
tấm thẻ sao cho tổng các số ghi trên các tấm thẻ đó bằng k, và chỉ có duy nhất một
cách chọn như vậy (nếu các tấm thẻ với các số giống nhau được coi là giống nhau)
cho với mỗi k. Có bao nhiêu các tập hợp các tấm thẻ như vậy?

n
11. Trong một cuộc thi toán, huy chương vàng được trao cho   người, huy
a
n n
chương bạc được trao cho   người và huy chương đồng được trao cho  
b  c
người. (a  b  c là các hằng số nguyên và n là số thí sinh tham dự). Không có ai
được nhật 2 hoặc nhiều hơn huy chương. Tìm tất cả các bộ ba số (a, b, c) với tính
chất sau.
Tính chất: Với mọi số nguyên k, tồn tại duy nhất 2 giá trị n sao cho số người
-không được huy chương đúng bằng k.
 [r] ký hiệu số nguyên lớn nhất không vượt quá r.

12. There is a village with a population 2007. This village has no name. You are
God of this village and you wants villagers to decide the name of this village.
Every villager has one idea of the village’s name.
Each villager can send a letter to each villager (including himself). And
every villager can send any number of letters every day. Letters are collected in the
evening and delevered at once the next morning every day. The Villager whoe
sends the letter can decide to whom the letter should be delivered. And each
vilagger can send a letter to tell the idea of the name of the village to God only one
time. This idea doesn’t need to be the same as the idea which he and the other
villagers had thuoght at first. And evey villager’s action is only writing a letter.
Every villager can be classified into an honest person or a liar. You and
every villager don’t know who is honest person, and who is a liar. But you know
that the number of liars is less than or equal to T, and there is one honest person at
least in this village.
You can give instruction to every villager only one at noon of one day. An
honest person necessarily follows the instruction, but you don’t know if a liar
follows the instruction. Find the maximal T which there exists an instruction
which fulfill the condition below.
 At last, every honest person sends a letter to God and every honest person
sends the same idea of the village’s name.
 If every honest person had thought the same idea of the name of the village
at first, every honest person sends this idea to God.

Đáp số
1. 18 2. 2 3. 21 4. 1352, 1734 5. 14 6. 1596 7. 402 8. 74 9. 8 10. 20
11. (6, 6, 6), (8, 8, 4), (10, 5, 5), (12, 6, 4) 12. 668

Hướng dẫn và lời giản ngắn gọn

1. Hãy xét thêm điểm M là trung điểm của AD.

2. 11n  10n + 1 mod 100.

3. Xét diện tích tam giác APB tính bằng 2 cách.

4. Giả sử A, B là các số có hai chữ số đầu và cuối của n. Ta cần tìm (A, B) sao cho
AB là ước của 100A + B.
Vì A chia hết 100A + B, ta phải có A chia hết B. Đặt k = B/A. Vì A, B là
các số có hai chữ số, ta có 10  A < 100/k.
Điều kiện tương đương với kA2 | 100A + kA hay kA | 100 + k. k chia hết
100 + k khi và chỉ khi k chia hết 100, và với k < 10, ta được k = 1, 2, 4, 5. Từ A |
(100 + k)/k và 10  A < 100/k ta được (k, A) = (2, 17), (4, 13) và n = 1734, 1352.

5. Chứng minh bằng quy nạp rằng n hình chữ nhật chia mặt phẳng thành nhiều
nhất 2n2 – 2n + 2 miền.

6. Xét bài toán tổng quát với các tấm thẻ 1, 2, …, n. Gọi F n là số các cách chọn khi
ta có n tấm thẻ. Dễ thấy rằng F 1 = 1 , F2 = 2. Giả sử k  3. Ta xét Fk. Nếu tấm thẻ k
không được chọn, số cách chọn hiển nhiên là Fk-1.
Xét trường hợp tấm thẻ k được chọn. Nếu mọi tấm thẻ nhỏ hơn k được
chọn, thì ta không thể chọn tấm thẻ 1. Và nếu ta bỏ đi tấm thẻ k và giảm các số
viết trên các tấm thẻ đi 1 đơn vị, ta có được một cách chọn thích hợp cho 1, 2, …,
k-2. Mặt khác, nếu ta có một cách chọn thích hợp từ 1, 2, …, k-2, ta thu được cách
chọn với tấm thẻ k và một số tấm thẻ khác từ 1, 2, …, k-1. Nếu không có tấm thẻ
nào ngoài tấm thẻ k được chọn thì hiển nhiên là chỉ có 1 cách. Vì vậy, số cách
chọn thích hợp với tấm thẻ k là Fk-2 + 1.
Vậy ta có hệ thức Fk = Fk-1 + Fk-2 + 1.
 Nếu ta cho phép cách chọn mà không có tấm thẻ nào, ta sẽ được dãy
Fibonacci.

7. Gọi f(n) là số cách biểu diễn số tự nhiên n dưới dạng tổng của các luỹ thừa
không âm của 3 và f(0) = 1. Sử dụng công thức truy hồi sau
f ( n)   f ( m)
0 m  n / 3

8. Gọi T1 là tứ diện có một mặt là ABC. Xét các trường hợp T 1 là ABCE, ABCF,
ABCG, ABCH.

9. Đầu tiên hãy chứng minh rằng a chia hết b.

10. Giả sử có k số khác nhau được viết trên các tấm thẻ. Giả sử a 1 < a2 < …< ak là
các số này và mỗi số được ghi trên p 1, p2, …, pk tấm thẻ tương ứng. Ta cần chứng
minh rằng (p1+1)(p2+1)…(pk+1) = 2008, a1 = 1 và ai = (p1+2)(p2+1)…(pi-1+1) (1 < i
 k).

11. Sử dụng bổ đề sau:


Bổ đề: Cho L và M là các số nguyên dương và g(n) là hàm số từ Z vào Z
sao cho g(n+tL) = g(n) + tM với mọi n, t. Giả sử g*(n) là số dư khi chia g(n) cho
M. Khi đó, với 0  k < m ta có khẳng định sau: nếu tồn tại đúng q giá trị của n với
0  n < L và g*(n) = k, với mọi số nguyên k’ đồng dư k theo mođun M tồn tại
đúng q giá trị n với g(n) = k’.
12. It’s very hard problem. But nice.

Vòng 2 (11/2/2007)

Thời gian làm bài: 4 giờ

Đề bài

1. Cho n là số nguyên dương. Hai người P và Q cùng chơi một trò chơi bằng cách
thay phiên nhau viết các số m (1  m  n). P viết số đầu tiên. Họ không được viết
những số đã được viết trước đó. Cuộc chơi sẽ kết thúc nếu ai đó không viết được
nữa. Nếu tổng các số được viết bởi P chia hết cho 3 thì P thắng, ngược lại Q thắng.
Tìm tất cả giá trị n thoả mãn điều kiện dưới đây:
Điều kiện: P sẽ thắng trò chơi này, không phụ thuộc vào cách chơi của Q.

2. Tìm tất cả các hàm số f, xác định trên tập các số thực dương và nhận các giá trị
thực sao cho
f ( x  y) f ( x) f ( y ) f ( x  y )
f ( x)  f ( y )  ,  
2 x y x y
với mọi số thực dương x và y.

3. Gọi () là đường tròn ngoại tiếp tam giác ABC. Gọi (A) là đường tròn tiếp xúc
với AB, AC và tiếp xúc ngoài với (), (B) là đường tròn tiếp xúc với BA, BC và
tiếp xúc ngoài với (), và (C) là đường tròn tiếp xúc CA, CB và tiếp xúx ngoài
với (). Giả sử (A), (B), (C) tiếp xúc () tại các điểm P, Q, R. Chứng minh rằng
các đường thẳng AP, BQ, CR đồng quy tại một điểm.

4. Trên mặt phẳng, dải chiều rộng d là tập tất cả các điểm có khoảng cách đến một
đường thẳng đã cho không vượt quá d/2. Có bốn điểm A, B, C, D trên mặt phẳng.
Nếu người ta chọn ba điểm bất kỳ trong các điểm này, tồn tại một dải độ dài 1
chứa chúng. Chứng minh rằng tồn tại một dải độ dài 2 chứa tất cả các điểm.

5. Ký hiệu [r] là số nguyên lớn nhất không vượt quá số thực r. Với số thực dương
x, gọi A(x) là tập hợp xác định bởi
A(x) = { [nx] | n là số nguyên dương}
Tìm tất cả các số vô tỷ  thoả mãn điều kiện sau.
Điều kiện: Nếu số thực dương  thoả mãn điều kiện A()  A() thì / là
số nguyên.

Đáp số

1. P có chiến thuật thắng khi và chỉ khi n  0, 4, 5 (mod 6).


2. Hãy đặt g(x) = f(x)/x; đáp số: f(x) = ax2.

3. Gọi MA, MB, MC là trung điểm các cung BC, CA, AB không chứa A, B, C tương
ứng và lA, lB, lC là các tiếp tuyến của () tại MA, MB, MC. Khi đó, để ý rằng lA//BC,
lB//CA, lC//AB. Sau đó hãy xét phép vị tự T tâm P biến (A) thành (), biến A thành
D. Tương tự xét phép vị tự tâm Q biến (B) thành (), biến B thành E và phép vị
tự tâm R biến (C) thành (), biến C thành F. Cuối cùng, hãy chứng minh rằng tồn
tại phép vị tự S biến A thành D, biến B thành E và biến C thành F. Tâm G của
phép vị tự này chính là điểm chung của ba đường thẳng AP, BQ, CR.

4. Sử dụng bổ đề sau.
Bổ đề. Cho tam giác XYZ, gọi HX là chân đường cao hạ từ X xuống YZ, H Y
là chân đường cao hạ từ Y xuống ZX và H Z là chân đường cao hạ từ Z xuống XY.
Nếu một dải chiều rộng w phủ tam giác XYZ thì min[XHX, YHY,ZHZ]  w.

5. Đáp số: Mọi số vô tỷ  > 2.

The 5th Japanese Junior Mathematical Olympiad (January 8, 2007)

Thời gian làm bài: 2 giờ.


Các bài 1, 2, 3, 4, 5 và 6 được 5 điểm. Các bài 7, 8, và 9 được 8 điểm.

Đề bài
1. Tìm giá trị của
7 5 7 5 4 7 5 4 3 7
        
12 12 11 12 11 10 12 11 10 9

2. Cho tứ giác ABCD thoả mãn điều kiện AB = 5, BC = 7, CD = 6. Biết rằng các
đường chéo AC và BD vuông góc với nhau. Hãy tính độ dài AD.

3. Tìm tất cả các cặp số nguyên dương (m, n) thoả mãn điều kiện
(1) m, n  20
(2) m và n nguyên tố cùng nhau
5 m 3
(3)  
7 n 4
4. Tìm một đa thức f(x, y, z) với hệ số thực, có bậc 3 thoả mãn các điều kiện sau.
* f(x, y, z) + x chia hết cho y+z
* f(x, y, z) + y chia hết cho z +x
* f(x, y, z) + z chia hết cho x + y
Đa thức P(x, y, z) chia hết cho đa thức Q(x, y, z) nghĩa là tồn tại đa thức R(x, y, z)
sao cho P(x, y, z) = Q(x, y, z).R(x, y, z).

5. Cho bảng vuông 5 x 5. Viết các số 1, 2, …, 16 (mỗi một số chỉ được viết 1 lần)
vào bảng 4 x 4 góc trên bên trái (*). Đối với 4 hàng, ta viết tổng của các số ở trên
mỗi hàng vào ô bên phải của mổi hàng. Tương tự, đối với 4 cột, ta viết tổng các số
trên mỗi cột vào ô cuống cùng của mỗi hàng. Và ta không viết gì ở ô dưới cùng
bên phải. Tìm số nguyên m lớn nhất thoả mãn điều kiện sau.
Điều kiện: Đối với bước (*), tồn tại một cách viết các số sao cho ta có thể
tìm được 2 số a, b với |a – b|  m đúng từ cột tận cùng bên phải cũng như từ hàng
tận cùng bên dưới.

6. Cho tam giác ABC và D, E, F là trung điểm của BC, CA, AB tương ứng. Nếu
AD = 3, BE = 4 và CF = 5 thì diện tích tam giác ABC bằng bao nhiêu?

7. Hai đường chéo AC và BD của tứ giác ABCD cắt nhau tại điểm P nằm trong tứ
giác. Nếu ta có AC = 2, BD = 3 và APB = 600, thì giá trị nhỏ nhất của AB + BC
+ CD + DA bằng bao nhiêu?

8. Giống bài 4 của vòng 1.


9. Ta có bảng vuông 19 x 19. Ta gọi một hình chữ nhật là tốt nếu cạnh của nó chạy
theo bờ của các hình vuông nhỏ. Tìm số n nhỏ nhất với tính chất sau.
Tính chất: Nếu 9 hình vuông nhỏ bị xoá đi khỏi bảng vuông, ta có thể chia
phần diện tích còn lại thành n hay nhỏ hơn các hình chữ nhật tốt.

Đáp số
1. 98/99 2. 2 3 3. 13/18, 8/11, 11/15, 14/194. k(x+y)(y+z)(x+y) – x –
y – z (k  0) 5. 35 6. 8 7. 7  19 8. 1352, 1734 9. 28
Selected problems from Thai Mathematical Olympiad Examination 2006

1. Giả sử rằng f: R  R là hàm số thoả mãn điều kiện


f(x2 + x + 3) + 2f(x2 – 3x + 5) = 6x2 – 10x + 17
với mọi x. Hãy tìm f(85).
8000
2. Hãy tính C
k 84
84
k
84
C 8084 k .

3. Tìm tất cả các giá trị n sao cho n2 + 59n + 881 là bình phương đúng.

4. Tìm số nguyên dương n nhỏ nhất sao cho phương trình


3 z n 1  z n  1  0
có nghiệm nằm trên đường tròn đơn vị.
2550

5. Gọi pk là số nguyên tố thứ k. Hãy tìm số dư khi chia p


k 2
p k4 1
k cho 2550.
6. Tìm tất cả các hàm số f: R  R sao cho
2005 2006 2006

 f (x
i 1
i  xi 1 )  f (  xi ) 
i 1
 f (2 x )
i 1
i
với mọi số thực x1, x2, …, x2006.

7. Cho tam giác với chu vi 2s. Nếu r là bán kính đường tròn nội tiếp và sa, sb, sc
là khoảng cách từ tâm đường tròn ngoại tiếp đến các đỉnh, chứng minh rằng
3 r r r s2
    .
4 s a s b s c 12r 2

8. Cho N là tập hợp các số nguyên dương. Tồn tại hay không song ánh f trên N
thoả mãn tất cả các điều kiện sau?
i) f(n+2006) = f(n) + 2006 với mọi n thuộc N
ii) f(f(n)) = n + 2 nếu n = 1, 2, 3, …, 2004
iii) f(2549) > 2550.

2 p 1  1
9. Tìm tất cả các số nguyên tố p sao cho p
là số chính phương.
10. 229 học sinh nam và 271 học sinh nữ được chia thành 10 nhóm, mỗi nhóm 50
học sinh được đánh số từ 1 đến 50. Người ta muốn chọn ra một nhóm 4 học sinh,
trong đó số học sinh nữ được chọn là lẻ và thoả mãn điều kiện sau đây: 4 người
này được chọn từ 2 nhóm và có 2 cặp học sinh có cùng số thứ tự. Chứng minh
rằng số cách chọn các nhóm như vậy là một số lẻ.

11. Tìm tất cả các hàm số f: R  R sao cho


f(x + cos(2007y)) = f(x) + 2007cos(f(y))
với mọi số thực x, y.

Đáp số, hướng dẫn

1. 167. Hãy thay x bằng 1-x.


169
2. C8085 . Xét bài toán chọn 169 số từ các phần tử của S = {0, 1, 2, …, 8084} và k
là trung vị của 169 số. Khi đó k có giá trị từ 84 đến 8000.

3. n = -40, n = -19.

4. n = 10.

5. 1273. Hãy chứng minh rằng nếu pk khác 2, 3, 5, 17 thì số hạng tương ứng trong
tổng đã cho cho số dư 1 khi chia cho 2550.

6. f là hàm hằng.
r A a
7. Sử dụng đánh giá s  sin 2  .
a 2 bc

8. Tồn tại. Định nghĩa hàm f như sau


n  1 if n  2,4,...,2006

f (n)  n  3 if n  1,3,...,2003
2 if n  2005

và f(n) = f(n-2006) + 2006 với mọi n > 2006.

9. p = 3, p = 7. Hãy chứng minh rằng với p > 7 thì (2 p-1-1)/p không thể là số chính
phương (phản chứng).

10. Gọi tập hợp gồm 4 học sinh lấy từ hai nhóm với hai học sinh có cùng số thứ tự
là một đội. Giả sử S = { | là một đội}, O = {  S|  có số lẻ học sinh nữ} và E
= {  S|  có số chẵn học sinh nữ}. Ta cần chứng minh rằng |O| là lẻ. Với mỗi A
 S, ta định nghĩa f ( A)  
A
g ( ) , trong đó g() là số học sinh nữ của .

Vì O  E =  và O  E = S, f(S) = f(O) + f(E).


Vì f(E) chẵn, f(S)  f(O) (mod 2).

f(S) có thể tìm được bằng cách xét rằng mỗi một học sinh nữ có thể bắt cặp với
một học sinh khác trong nhóm bởi 50 – 1 cách, sau đó tìm 2 học sinh khác ở nhóm
khác bởi 10 – 1 cách. Suy ra, học sinh nữ này là thành viên của 49.9 đội. Có nghĩa
là mỗi một học sinh nữ được tính 49.9 lần trong f(S). Vì ta có 271 học sinh nữ,
f(S) = 49.9.271  1 (mod 2).

Vì mỗi   O chứa một số số lẻ các học sinh nữ, như thế f(O)  |O| (mod 2). Suy
ra |O|  f(O)  f(S)  1 (mod 2) và như vậy số cách chọn những đội như vậy là một
số lẻ.

11. Cho x = 0 vào phuơng trình hàm, ta có


f(cos(2007y)) = f(0) + 2007cos(f(y)) (*)
Định nghĩa g: R  R bởi g(x) = f(x) – f(0) với mọi x thuộc R. Khi đó
g(x + cos2007y) = g(x) + g(cos2007y) với mọi y thuộc R.
Bây giờ đặt z = cos2007y thì
g(x+z) = g(x) + g(z) với mọi z  [-1, 1]
từ đó, có thể mở rộng thành
g(x+nz) = g(x) + g(nz)
với mọi z thuộc [-1, 1] và với mọi số nguyên dương n. Với mỗi y thuộc R, ta tìm
được số nguyên dương n sao cho y/n thuộc [-1, 1], từ đó suy ra g thoả mãn phương
trình hàm Cauchy trên R. Mặt khác, từ (*) ta suy ra f bị chặn trên (-1, 1), từ đó suy
ra g cũng bị chặn.

Như vậy, g là hàm tuyến tính. Chú ý rằng theo định nghĩa thì g(0) = 0; từ đó g(x) =
ax với một hằng số a nào đó.

Thay ngược trở lại vào phương trình và tìm a và f(0) ta được 3 lời giải:
f(x) = (k+1/2),
f(x) = 2007x + 2k, hay
f(x) = -2007x + (2k+1).
với k là số nguyên tuỳ ý.

14th Turkish Mathematical Olympiad, 2006

Ngày thứ nhất

Bài 1. Cho E và F là hai điểm trên cạnh CD của tứ giác lồi ABCD thỏa mãn 0 <
DE = FC < CD. Gọi K là giao điểm thứ hai của các đường tròn ngoại tiếp tam giác
ADE và ACF, L là giao điểm thứ hai của các đường tròn ngoại tiếp tam giác BDE
và BCF. Chứng tỏ rằng các điểm A, B, K, L nằm trên một đường tròn.

Gợi ý: Dùng phương tích

Bài 2. Trong một trường học gồm 2006 học sinh và 14 giáo viên, mỗi học sinh đều
biết ít nhất một giáo viên. Với mỗi học sinh A và một giáo viên B biết nhau, tỉ số
giữa số học sinh biết giáo viên B và số giáo viên mà học sinh A biết là t. Hãy tìm
giá trị lớn nhất của t.

Đáp số: tmax =2006/14.

Bài 3. Tìm tất cả các số nguyên dương n sao cho mọi hệ số của đa thức
đều chia hết cho 7.

Gợi ý: Sử dụng tính chất

với mọi đa thức Q(x) với hệ số nguyên.


Đáp số: n = 7k hoặc n = 7k + 7l với 0 ≤ k≤ l

Bài 4. Cho a1, a2, …, an là các số thực dương sao cho n ≥ 2 và


t = a1 + a2 + … + an = a12 + a22 + … + an2.
Chứng minh rằng

Gợi ý: Dùng BĐT Cauchy-Schwarz.

Bài 5. Gọi A1, B1, C1 lần lượt là chân đường cao hạ từ A, B, C của tam giác ABC
và OA, OB, OC lần lượt là tâm đường tròn nội tiếp tam giác AB 1C1, BC1A1, CA1B1.
Đặt TA, TB, TC lần lượt là tiếp điểm của đường tròn nội tiếp tam giác ABC với các
cạnh BC, CA, AB. Chứng tỏ rằng TAOCTBOATCOB là lục giác đều.

Bài 6. Chứng tỏ rằng không tồn tại một tam giác sao cho độ dài các cạnh, diện tích
và số đo các góc (theo đơn vị độ) đều là các số hữu tỷ.

Gợi ý: Áp dụng định lý hàm số sin và cosin và chứng tỏ sin và cosin của một góc
không thể cùng hữu tỷ.

Ngày thứ hai

Bài 1. Một hãng hàng không cần lên kế hoạch bay khứ hồi giữa các thành phố A,
B, C, D, E và F. Xác định số cách sắp xếp có thể được sao cho có thể đi lại giữa
hai thành phố bất kỳ trong 6 thành phố trên mà chỉ dùng các chuyến bay của hãng
hàng không đó.

Đáp số: 26704.

Bài 2. Cho A và B là hai điểm phân biệt trên đường tròn Γ. Gọi P là một điểm bất
kỳ trên đường tròn Γ và không trùng với các điểm A và B. Tìm quỹ tích của điểm
M sao cho PM là tia đối của phân giác APB và MP = AP + PB.
Đáp số: Quỹ tích điểm M là hợp của 2 cung tròn chứa A và B (có tâm thuộc
đường trung trực đoạn AB).

Bài 3. Chứng tỏ rằng

với mọi số thực dương a, b, c thỏa mãn đẳng thức a + b + c = 1.

Bài 4. Các tam giác nhọn ABC và A1B1C1 là đồng dạng sao cho các đỉnh B1, C1 và
A1 lần lượt nằm trên các tia AC, BA và CB. Chứng tỏ rằng trực tâm của tam giác
A1B1C1 và tâm đường tròn ngoại tiếp tam giác ABC là trùng nhau.

Bài 5. Tìm tất cả các số nguyên dương lẻ n sao cho tồn tại các số nguyên lẻ x 1, x2,
…, xn thỏa mãn điều kiện x12 + x22 + … + xn2 = n4.

Đáp số: n = 8k + 1.

Bài 6. Xác định số cách gán các số 1 và 1 vào các ô của hình vuông gồm
20072007 ô sao cho tổng của tất cả các số trong một hình vuông bất kỳ (được tạo
từ các ô) có giá trị tuyệt đối không vượt quá 1.

Đáp số: 21006  6.

Estonian Math Competitions - IMO Team Selection Contest

First Day

Bài 1. Trong một mặt phẳng, cho một đường thẳng ℓ và hai đường tròn c 1 và c2
không cùng bán kính sao cho ℓ tiếp xúc với cả hai đường tròn tại điểm P. Gọi M ≠
P là một điểm nằm trên ℓ và Q1, Q2 lần lượt là tiếp điểm của tiếp tuyến (khác ℓ) kẻ
từ M đến c1 và c2. Tìm PMQ1 + PMQ2.

Đáp án: π.
Hướng dẫn: Xét trường hợp Q1 và Q2 nằm cùng phía và khác phía so với ℓ.

Bài 2. Trên hành tinh Automory có rất nhiều người sinh sống. Mỗi người trong
hành tinh đều yêu đúng một người và kính trọng đúng một người. Ngoài ra, còn có
các thông tin sau:
- Mỗi người đều được một người nào đó yêu;
- Nếu người A yêu người B thì mọi người kính trọng A đều yêu B;
- Nếu người A kính trọng người B thì mọi người yêu A đều kính trọng B.
Đúng hay sai nếu khẳng định rằng mọi người trong hành tinh Automory đều yêu
và kính trọng cùng một người?

Đáp án: đúng.

Bài 3. Tìm tất cả các cặp số nguyên dương (x, y) thỏa mãn phương trình (x + y) x =
xy.

Đáp án: (2, 6) và (3, 6).

Second Day

Bài 4. Tìm tất cả các cặp số thực (a, b) sao cho các nghiệm của đa thức 6x 2  24x
 4a và x3 + ax2 + bx  8 là các số thực không âm.

Đáp án: ( 6, 12).

Bài 5. Trên một đường thẳng nằm ngang, cho 2005 điểm được đánh dấu trắng
hoặc đen. Với mỗi điểm, xác định tổng tất cả các điểm trắng bên phải và điểm đen
bên trái của nó. Biết rằng, trong 2005 tổng trên có đúng một số xuất hiện số lẻ lần.
Hãy tìm tất cả các giá trị có thể có của số này.

Đáp án: 1002.


Bài 6. Trong mặt phẳng, đường thẳng ℓ và một đường tròn không có điểm chung.
Gọi AB là đường kính của đường tròn vuông góc với ℓ, trong đó B gần ℓ hơn A.
Gọi C là một điểm trên đường tròn và khác hai điểm A, B. Đường thẳng AC cắt ℓ
tại D. Gọi E là tiếp điểm của tiếp tuyến kẻ từ D đến đường tròn sao cho B và E
nằm cùng phía so với AC. Đường thẳng EB cắt ℓ tại điểm F, đường thẳng FA cắt
đường tròn tại điểm G khác A. Chứng minh rằng điểm đối xứng với G qua AB
thuộc đường thẳng FC

Hướng dẫn: Xem IMO-2004 Shortlist.


British Mathematical Olympiad-2007

Voøng 1 – 1/12/2006

Baøi 1: Tìm 4 soá nguyeân toá nhoû hôn 100 laø öôùc soá cuûa
332 – 232.

Baøi 2: Trong töù giaùc loài ABCD, caùc ñieåm M, N naèm treân
caïnh AB sao cho AM= MN= NB, vaø caùc ñieåm P, Q naèm treân
CD sao cho CP= PQ= QD. Chöùng minh raèng:
SAMCP = SMNPQ = (1/3). SABCD .

Baøi 3: Soá 916238457 laø moät ví duï veà moät soá coù 9
chöõ soá maø trong ñoù caùi chöõ soá 1,2,…,9 xuaát hieän
ñuùng moät laàn. Noù coøn theâm coù tính chaát caùc chöõ soá
1,2,3,4,5 ñöôïc xeáp theo ñuùng thöù töï tö nhieân cuûa noù,
nhöng caùc chöõ soá 1,2,3,4,5,6 khoâng xeáp theo ñuùng thöù
töï nhieân cuûa noù. Coù bao nhieâu soá nhö vaäy ?
Baøi 4: Cho hai ñöôøng troøn S vaø T tieáp xuùc nhau. A
thuoäc S, B thuoäc T sao cho AB laø tieáp tuyeán chung cuûa hai
ñöôøng troøn. Giaû söû AP laø ñöôøng kính cuûa voøng troøn S,
töø P keû tieáp tuyeán PQ cuûa ñöôøng troøn T. Chöùng minh
raèng AP = PQ.

Baøi 5: Cho a, b, c laø caùc soá thöïc döông, chöùng minh baát
ñaúng thöùc sau:
(a2 + b2 )2 ≥ (a+b+c). (a +b-c). (b+c-a).(c+a-b).

Baøi 6: Cho n laø moät soá nguyeân. Chöùng minh raèng, neáu
2+2.(1+12.n2)1/2 laø moät soá nguyeân, thì noù laø moät soá
chính phöông.

Voøng 2 – 30/1/2007

Baøi 1: Cho tam giaùc ABC coù ñoä daøi caùc caïnh laø soá
nguyeân, vaø AC = 2007. Ñöôøng phaân giaùc goùc A cuûa tam
giaùc ABC caét BC ôû D.
Giaû söû AB = CD, xaùc ñònh ñoä daøi caïnh AB vaø caïnh BC.

Baøi 2: Chöùng minh raèng toàn taïi voâ haïn caëp soá (m,n)
nguyeân döông sao cho:
(m+1)/n + (n+1)/m
laø moät soá nguyeân döông.

Baøi 3: Cho tam giaùc ABC nhoïn vôùi AB > AC vaø goùc BAC =
60 . Goïi O laø taâm ñöôøng troøn ngoaïi tieáp, H laø tröïc taâm
o

tam giaùc ABC. Ñöôøng thaúng OH caét AB taïi P vaø caét AC taïi
Q. Chöùng minh raèng PO = HQ.

Baøi 4: Treân ñaûo Hexagonia, saùu thaønh phoá lieân thoâng


vôùi nhau baèng moät heä thoáng ñöôøng ray xe löûa sao cho
coù moät ñöôøng ray tröïc tieáp lieân thoâng vôùi moãi caëp
thaønh phoá. Vaøo nhöõng ngaøy chuû nhaät, moät soá ñöôøng
ray coù theå bò ñoùng laïi cho vieäc chuaån bò. Nhöõng haønh
khaùch xe löûa phaûi ñöôïc ñaùp öùng raèng baát kì thaønh phoá
naøo cuõng phaûi coù loä trình ñi ñeán thaønh phoá khaùc baèng
ñöôøng ray ( khoâng nhaát thieát laø tröïc tieáp) taïi taát caû
thôøi gian. Coù bao nhieâu caùch khaùc nhau coù theå coù maø
moät soá ñöôøng ray ñoùng cöûa nhöng vaãn thoûa maõn yeâu
caàu treân.
Solutions of BMO-2007

Voøng 1 – 1/12/2006

Baøi 1:
Deã thaáy:
232 – 332 = (316+216). (38+28). (34+24). (32+22). (3+2). (3-2)
Töø coâng thöùc naøy ta coù theå tìm ñöôïc caùc öôùc soâng
nguyeân toá thoûa ñeà baøi laø: 5, 13, 17, 97.

Baøi 2:
Ta coù : SAMCP = SAMC + SAPC = (1/3) SADC + (1/3). SACB = (1/3). SABCD.
SMNPQ = SMNP + SMPQ
Maët khaùc :
SMNP = SAMP ; SMPQ = SPCM
Do ñoù: SMNPQ = SAMCP = (1/3). SABCD .

Baøi 3:
Soá soá thoûa maõn tính chaát baøi toaùn: 2520

Baøi 4:
Goïi O1, O2 laàn löôït laø taâm ñöôøng troøn S, T
a, b laàn löôït laø baùn kính ñöôøng troøn S, T
Aùp duïng ñònh lyù Pytago ta coù:
AB2 = (a+b)2 - | a-b| 2 = 4.a.b
(PO2) 2 = | b-2.a|2 + AB2 = 4.a2 + b2
Suy ra: PQ2 = (PO2) 2 – (QO2)2 = 4.a2
Do ñoù PQ = 2.a = PA .

Baøi 5:
Neáu a, b, c khoâng laø 3 caïnh tam giaùc thì
(a+b+c). (a +b-c). (b+c-a).(c+a-b) ≤ 0 ≤ (a2 + b2 )2
Neáu a, b, c laø 3 caïnh cuûa moät tam giaùc ABC
Theo coâng thöùc heron ta coù:
(a+b+c). (a +b-c). (b+c-a).(c+a-b) = 16. (S ABC)2
Maët khaùc:
SABC = 1/2 . a.b sinC ≤ 1/2 . a.b ≤ 1/4 (a2 + b2)
Do ñoù:
(a+b+c). (a +b-c). (b+c-a).(c+a-b) ≤ (a2 + b2)2 .
Daáu “=” baát ñaúng thöùc xaûy ra khi vaø chæ khi a=b vaø c 2
= a2 + b2 .
Baøi 6:
Deã thaáy, neáu 2+2.(1+12.n2)1/2 laø moät soá nguyeân thì 1+
12.n2 laø soá chính phöông
Töø ñoù suy ra: 1+ 12.n2 = (1+2.k)2
Suy ra: 3. n2 = k. (k+1).
Vì k vaø k+1 laø hai soá nguyeân toá cuøng nhau neân ta coù 2
tröôøng hôïp sau
Tröôøng hôïp 1: k laø soá chính phöông vaø k+1 chia heát cho 3 .
Ñieàu naøy maâu thuaãn
Tröôøng hôïp 2: k+1 laø soá chính phöông
Ta coù: 2+2.(1+12.n2)1/2 = 4.(k+1) laø soá chính phöông.

Voøng 2 – 30/1/2007

Baøi 1:
Ñaët AB = CD = c, vaø BD = k, theo ñieàu kieän baøi toaùn ta
deã daøng suy ra c, k laø hai soá nguyeân döông.
Aùp duïng ñònh lyù ñöôøng phaân giaùc ta coù:
c/k = 2007/c
Suy ra c2 = 2007 . k = 32 . 223. k
Vì 223 laø soá nguyeân toá neân k = 223. b 2 vôùi b laø soá
nguyeân döông
Neáu b = 1, ta coù c = 669, k = 223 thì
AB + BC = 2.c + k = 1561 < 2007 = AC ,
Maâu thuaãn vôùi baát ñaúng thöùc tam giaùc.
Neáu b > 2
BC = k + c ≥ 223. 32 + c = AB + AC,
Maâu thuaãn vôùi baát ñaúéng thöùc tam giaùc.
Do ñoù b = 2, suy ra k = 892 vaø c = 1338
Suy ra AB = 1338 vaø BC = 2230 thoûa maõn baát ñaúng thöùc
tam giaùc
Vaäy AB = 1338 , BC =2230.
Baøi 2:

Nhaän xeùt: Neáu toàn taïi n, m, k laø 3 soá nguyeân döông


thoûa maõn hai ñieàu sau
m<n
(m+1)/n + (n+1)/m = k
Thì toàn taïi n , r , k laø 3 soá nguyeân döông cuõng thoûa maõn
n<r
(m+1)/r + (r+1)/m = k
Vôùi r = n.(n+1)/m .
Do ñoù neáu (m, n) laø nghieäm thì (n , r) cuõng laø nghieäm vaø
n+r > m+n
Ta coù (1 , 2 ) laø nghieäm
Vaäy toàn taïi voâ haïn caëp soá nguyeân döông (m , n) thoûa
maõn ñieàu kieän baøi toaùn.

Baøi 3:

Goïi N, E laàn löôït laø chaân ñöôøng vuoâng goùc töø O xuoáng
AB, töø B xuoáng AC.
Suy ra N laø trung ñieåm cuûa AB, do ñoù AN = ½ AB
Maët khaùc BAE = BAC = 60o , suy ra tam giaùc ABE laø nöûa
tam giaùc ñeàu . Töø ñoù suy ra AE = ½ AB
Do ñoù : AN = AE = ½ AB (1)
Ta coù : NAO = EHA = ACB (2)
Töø (1) vaø (2) suy ra hai tam giaùc vuoâng AON vaø AHE baèng
nhau .
Suy ra: NO = HE (3)
AO = AH, suy ra tam giaùc OAH caân taïi A. Vì theá
NOP= 1800 - ACB - AOH = 1800 - ACB - AHO = EHQ
(4)
Töø (3) vaø (4) suy ra hai tam giaùc vuoâng ONP vaø HEQ baèng
nhau
Vaäy OP = HQ.

Baøi 4: Baøi toaùn töông ñöông vôùi tìm soá ñoà thò lieân
thoâng treân 6 ñænh.
Goïi f(i) laø soá ñoà thò lieân thoâng treân n ñænh.
Goïi a(i) laø soá ñoà thò coù i ñænh
a(0) = 1
Ta coù: a(i) = 2i.(i-1)/2
Ta tìm moái lieân quan cuûa f(1), f(2), …, f(n) vôùi a(n)
Xeùt trong ñoà thò n ñænh coù caùch ñænh coá ñònh, ta coù a(n)
ñoà thò
Choïn moät ñænh v coá ñònh. Ta coù v naèm trong thaønh phaàn
lieân thoâng coù i ñænh vôùi i chaïy töø 1 ñeán n. Coøn laïi n-1
(n  1)!
ñænh, vaø vôùi moãi i coù ci  (i  1)!.(n  i )! caùch choïn i-1 ñænh
trong n-1 ñænh naèm trong thaønh phaàn lieân thoâng cuûa v.
Vôùi i-1 ñænh ñoù, coù f(i) caùch choïn thaønh phaàn lieân
thoâng cuûa v (theo ñònh nghóa f(i)).
Coøn laïi n-i ñænh coù theå noái vôùi nhau moät caùch baát kì
vaø coù a(i) caùch.
Do ñoù ta coù coâng thöùc sau:
a(n) = c1.f(1).a(n-1)+ c2.f(2).a(n-2)+…+ cn.f(n).a(n-n).
Döïa vaøo coâng thöùc treân vaø baét ñaàu tính töø f(1) =1 , f(2)
= 1, ta coù theå tính ñöôïc:
f(3)=4 , f(4) = 38 , f(5) = 728 vaø f(6) = 26704 .
Vaäy soá caùch thoûa maõn ñeà baøi laø 26704.

38th Austrian Mathematical Olympiad

Regional competition (qualifying Round) – 24/6/2007

Baøi 1: Cho 0 < x1, x2, …, x669 < 1 laø caùc soá thöïc khaùc
nhau töøng ñoâi moät. Chöùng minh toàn taïi moät caëp soá (x i ,
xj) sao cho
0 < xi . xj . ( xj – xi ) < 1/2007
Baøi 2: Tìm taát caû caùc boä 5 soá nguyeân döông ( x 1, x2, x3,
x4, x5 ) vôùi x1>x2> x3> x4> x5 sao cho
[(x1+x2)/3]2 +[(x2+x3)/3]2 +[(x3+x4)/3]2 +[(x4+x5)/3]2 = 38
Vôùi [x] laø phaàn nguyeân cuûa x.

Baøi 3: So saùnh giaù trò cuûa S vaø T :


2. n 1
( k  1) 2 2.n 1
k2
S 
k  2. n
k 
T  
k  2.n
k 
2 2
   
a a
Vôùi [x] laø phaàn nguyeân cuûa x, | y | laø giaù trò tuyeät ñoái
cuûa y.

Baøi 4: Cho töù giaùc loài ABCD, M laø giao ñieåm 2 ñöôøng
cheùo. Xaùc ñònh taát caû caùc töù giaùc ABCD sao cho toàn taïi
moät ñöôøng thaúng g qua M caét AB, CD laàn löôït ôû P, Q thoûa
maõn tính chaát: 4 tam giaùc APM, BPM, CQM vaø DQM ñoàng
daïng

National competition (Final round, part 1): 17/5/2007

Baøi 1: Cho moät baûng 2007 x 2007. Moät soá nguyeân leû
ñöôïc vieát vaøo moãi oâ cuûa noù. Zi laø toång soá ôû doøng
thöù i vaø Sj laø toång soá ôû coät thöù j vôùi 1≤ i, j ≤ 2007. Ta
ñaët :
A = Z1.Z2… Z2007 , B = S1.S2…S2007 .
Chöùng minh raèng A+B khaùc 0 .

Baøi 2: Tìm giaù trò lôùn nhaát cuûa C(n) cho taát caû soâ
nguyeân döông n, sao cho:
(n+1).[ (a1)2+(a2)2+…+(an)2] – (a1+a2+…+an)2 ≥
C(n)
Vôùi taát caû boä n soá (a1,a2,…,an) khaùc nhau töøng ñoâi moät.

Baøi 3: Cho M(n) = {-1, -2, …, -n }. Vôùi moãi taäp con khaùc
roãng cuûa M(n), ta ñònh nghóa tích cuûa caùc phaàn töû cuûa
noù. Tính toång cuûa caùc tích .

Baøi 4: Cho n > 4 laø moät soá nguyeân. Xeùt moät n giaùc loài
noäi tieáp A0A1…An-1An ( vôùi A0 = An ) sao cho Ai-1Ai = i vôùi 1 ≤ i
≤ n . Hôn nöõa, ai laø goùc giöõa ñöôøng thaúng A iAi+1 vaø tieáp
tuyeán cuûa ñöôøng troøn ngoaïi tieáp n giaùc taïi A i (theo goùc
nhoïn) . Tính giaù trò cuûa
P = a0 + a1 + … + an-1 .

National competition (Final round, part 2, first day): 5/6/2007

Baøi 1: Tìm taát caû soá nguyeân khoâng aâm a < 2007 sao
cho phöông trình
x2 + a ≡ 0 ( mod 2007)
coù ñuùng 2 nghieäm nguyeân khoâng aâm nhoû hôn 2007.
Baøi 2: Tìm taát caû caùc boä 6 soá (x 1, x2, x3, x4, x5, x6)
nguyeân khoâng aâm thoûa maõn heä phöông trình sau:
x1.x2.(1-x3) = x4.x5
x2.x3.(1-x4) = x5.x6
x3.x4.(1-x5) = x6.x1
x4.x5.(1-x6) = x1.x2
x5.x6.(1-x1) = x2.x3
x6.x1.(1-x2) = x3.x4
Baøi 3: Tìm taát caû hình thoi coù caïnh laø 2.a sao cho toàn
taïi moät ñöôøng troøn thoûa maõn tính chaát: Moãi moät caïnh
cuûa hình thoi giao vôùi ñöôøng troøn taïo thaøn moät daây cung
coù ñoä daøi laø a.

National competition (Final round, part 2, second day): 6/6/2007

Baøi 4: Cho M laø taäp hôïp caùc ña thöùc P(x) heä soá
nguyeân coù caùc nghieäm nguyeân khaùc nhau töøng ñoâi
moät vaø trò tuyeät ñoái cuûa caùc heä soá ñeàu nhoû hôn
2007.
Baäc cao nhaát cuûa moät ña thöùc trong M coù theå
laø bao nhieâu ?

Baøi 5: Cho moät n giaùc loài vôùi moät moät pheùp ñaïc giaùc,
coù nghóa laø moät pheùp phaân chia thaønh nhöõng tam giaùc
bôûi nhöõng cheùo khoâng caét nhau. Chöùng minh raèng n goùc
cuûa n giaùc coù theå ñöôïc ñaùnh soá bôûi nhöõng chöõ soá
cuûa 2007 sao cho baát kì töù giaùc naøo goàm coù 2 tam giaùc
trong pheùp ñaïc giaùc vôùi moät caïnh chung coù goùc ñöôïc
ñaùnh soá vôùi toång baèng 9.
Baøi 6: Cho tam giaùc ABC vôùi ñöôøng troøn ngoaïi tieáp U.
Moät ñieåm P treân tia ñoái tia AU. Goïi g laø ñöôøng thaúng ñoái
xöùng vôùi PB qua BA vaø h laø ñöôøng thaúng ñoái xöùng vôùi
PC qua CA. Goïi G laø giao ñieåm cuûa g vaø h.
Tìm quó tích cuûa Q khi P thay ñoåi treân tia ñoái tia AU.
The 20th Korean Mathematical Olympiad (KMO)

Voøng 1 – 24/3/2007

Baøi 1: Cho tam giaùc ABC nhoïn , O laø taâm ñöôøng troøn
ngoaïi tieáp tam giaùc. Ñöôøng troøn taâm O’ tieáp xuùc vôùi
ñöôøng troøn O taïi A vaø tieáp xuùc vôùi caïnh BC taïi D. Goïi E,
F, A’(A ) vaø G(E) laàn löôït laø giao ñieåm cuûa ñöôøng
thaúng AB, AC, OO’, EO’ vôùi ñöôøng troøn (O’). Ñöôøng thaúng
BO vaø A’G giao nhau taïi H.
Chöùng minh raèng DF2 = AF . GH.

Baøi 2: Cho 16 vieân ñaù laùt coá ñònh treân töôøng nhöng
hình veõ sau. Coù bao nhieâu caùch ñeå vieát soá 0 hoaëc 1
treân moãi vieân sao cho tích hai soá ñöôïc vieát treân moãi caëp
vieân keà nhau ( chung caïnh ) luoân baèng 0 ?

Baøi 3: Tìm taát caû boä ba soá ( x, y, z ) nguyeân döông thoûa


maõn: 1 + 4x + 4y = z2.

Voøng 2 – 25/3/2007

Baøi 4: Tìm taát caû caùc caëp soá nguyeân toá (p,q) sao cho:
pp + qq + 1 chia heát cho p.q

Baøi 5: A laø ñænh cuûa tam giaùc ABC, goïi l A laø khoaûng
caùch cuûa hai ñieåm laø chaân ñöôøng vuoâng goùc ñeán caïnh
AB vaø BC töø giao ñieåm phaân giaùc goùc A vôùi caïnh BC.
Töông töï, lB, lC cho ñænh B, C. Chöùng minh baát ñaúng thöùc
sau:
lA. lB. lC ≥ (1/64). l3
vôùi l laø chu vi cuûa tam giaùc ABC.

Baøi 6: Cho haøm soá f: N → N thoûa tính chaát sau:


k. f(n) ≤ f(k.n) ≤ k. f(n) + k -1
vôùi moãi k, n thuoäc N, vôùi N laø taäp hôïp caùc soá nguyeân
döông.
Chöùng minh:
a) Vôùi moïi a,b thuoäc N,
f(a) + f(b) ≤ f(a+b) ≤ f(a) + f(b) + 1
b) Neáu f thoûa maõn
f(2007) ≤ 2007. f(n) + 2005
vôùi moãi n thuoäc N thì toàn taïi c thuoäc N sao cho : f(2007.c)
= 2007. f(c)

Solutions of the 20th KMO

Voøng 1 – 24/3/2007

Baøi 1:
Goïi X laø giao ñieåm cuûa BO vôùi ñöôøng troøn O.
Ta chöùng minh ñöôïc:
A, G, X thaúng haøng.
EG song song vôùi BH.
Maët khaùc: AA’, EG laø ñöôøng kính cuûa O’ neân AEA’G laø
hình chöõ nhaät. Töø ñoù suy ra:
EG = BH. (1)
O vaø O’ tieáp xuùc taïi A ta coù : EF song song vôùi BC
Töø ñoù suy ra : DF = DE (2)
Tam giaùc ABD ñoàng daïng vôùi tam giaùc ADF ta coù:
AF/AD = DF/BD (3)
Tam giaùc ABD ñoàng daïng vôùi tam giaùc BDE ta coù:
DE/AD = BE/BD (4)
Töø (1), (2), (3), (4) suy ra: DF2 = AF . GH

Baøi 2:
Khi vieát 0,1 treân moãi vieân gaïch thoûa maõn tính chaát tích 2
soá ñöôïc vieát keà nhau treân vieân gaïch laø 0. Ta goïi moãi
traïng thaùi treân laø moät z-pattern.
Boå ñeà 1:

Goïi an laø soá z-pattern cho n vieân gaïch naèm treân moät
doøng.

Ta chöùng minh deã daøng coâng thöùc sau:


an = Fn+2 vôùi moïi n ≥ 1
Fn laø soá Fibonacci thöù n trong daõy soá Fibonacci
F1 = 1, F2 = 1
Fn+2 = Fn+1 + Fn .
Boå ñeà 2:

Goïi bn laø soá z-pattern cho n vieân gaïch naèm treân moät
hình voøng nhö sau:

Ta coù coâng thöùc sau:


bn = an-1 + an-3 = Fn+1 + Fn-1 vôùi moïi n ≥ 4 .

Quay laïi baøi toaùn:


Caùch ñieàn soá xuaát phaùt töø hình vuoâng 2 x2 ôû chính
giöõa neàn gaïch 16 oâ
Xeùt hình vuoâng 2 x 2 chính giöõa treân neàn gaïch 16 oâ nhö
hình veõ

Ta coù 3 tröôøng hôïp sau:


Tröôøng hôïp 1: 4 soá 0
0 0
0 0
Söû duïng boå ñeà 2 cho 12 vieân gaïch coøn laïi. Ta coù soá z-
pattern laø:
b12 = F13 + F11 = 233 + 89 = 322
Tröôøng hôïp 2: 3 soá 0
1 0 0 1 0 0 0 0
0 0 0 0 1 0 0 1

Caùc tröôøng hôïp treân seõ coù soá z-pattern nhö nhau, xeùt
tröôøng hôïp ñaàu tieân. Theo ñieàu kieän baøi toaùn thì caùc
soá phaûi ñieàn tieáp theo nhö sau:
0
0 1 0
0 0

Coøn laïi 10 vieân gaïch, vieân gaïch treân cuøng coù theå ñieàn
0 hoaëc 1 tuøy yù. Coøn laïi 9 vieân, thì soá caùch ñieàn chính
laø soá z-pattern cho 9 vieân gaïch naèm ngang. Do ñoù, soá z-
pattern cho tröôøng hôïp naøy laø: 2. a9 = 2. F11 = 2. 89 = 178
Töø ñoù suy ra soá z- pattern cho tröôøng hôïp 2 laø: 4. 178 =
712 .
Tröôøng hôïp 3: 2 soá 0

1 0 0 1
0 1 1 0
2 tröôøng hôïp naøy coù soá z-pattern nhö
nhau:
Xeùt tröôøng hôïp ñaàu:
0
0 1 0
0 1 0
0
2 vieân gaïch ôû goùc coù theå ñieàn 0 hoaëc 1 tuøy yù. Ta coù 4
caùch ñieàn
Coøn laïi 6 vieân chia laøm 2 mieàn. Thì soá z pattern trong töøng
mieàn laø: a3 = F5 .
Do ñoù soá z-pattern trong tröôøng hôïp naøy laø: 4. F 5 . F5 = 4 .
5 . 5 = 100
Suy ra soá z- pattern trong tröôøng hôïp 3 laø : 2 .100 = 200 .
Vaäy soá caùch ñieàn thoûa maõn ñeà baøi laø : 322 + 712 +
200 = 1234.

Baøi 3:
Khoâng maát tính toång quaùt, ta giaû söû x ≤ y
Neáu 2.x < y+1 thì
(2y)2 < 1 + 4x + 4y < 1 + 2y+1 + 4y = (1 + 2y)2 .
Neáu 2.x = y+1 thì
1 + 4x + 4y = (1 + 2y)2
Do ñoù: (x,y,z) = (x, 2.x-1, 1+2 2.x-1)
Laø nghieäm cuûa phöông trình.
Neáu 2.x > y+1, suy ra x > 1
Phöông trình töông ñöông: 4x + 4y = 4x .(1 + 4y-x ) = (z-1). (z+1)
Vì öôùc soá chung lôùn nhaát cuûa (z-1) vaø (z+1) laø 2 neân z-1
hoaëc z +1 chia heát cho 22.x-1
Ñieàu naøy maâu thuaãn vì
2. (1+ 4y-x) ≤ 2. (1+ 4x-2) ≤ 22.x-1 -2 vôùi
moïi x > 1.
Vaäy boä 3 soá thoûa maõn ñeà baøi laø:
(x, 2.x-1, 1+22.x-1) hoaëc (2.x-1, x, 1+22.x-1) .

Voøng 2 – 25/3/2007

Baøi 4:
Deã thaáy pq. Khoâng maát tính toång quaùt ta giaû söû p < q .
Neáu p=2 . Ta coù
qq + 5 ≡ 5 ≡ 0( mod q )
suy ra, q=5.
Neáu p  2 thì p vaø q ñeàu laø soá nguyeân toá leû. Hôn nöõa, q
– p ≥ 2 (1)
Töø ñieàu kieän baøi toaùn ta suy ra : pp + 1 ≡ 0 ( mod q )
Suy ra: p2.p ≡ 1 ( mod q)
Theo ñònh lyù fermat nhoû ta coù : pq-1 ≡ 1 ( mod q )
Do ñoù : p(2.p, q-1) ≡ 1 ( mod q )
Ta coù: (2.p, q-1) = 2 hoaëc (2.p, q-1) = 2.p

Neáu (2.p, q-1) = 2 thì p2 ≡ 1 ( mod q )


Suy ra: p ≡ 1 ( mod q ) hoaëc p ≡ -1 ( mod q )
Ñieàu naøy maâu thuaãn vôùi (1)
Neáu (2.p, q-1) = 2.p thì q ≡ 1 ( mod p )
Suy ra: 1 + pp + qq ≡ 1 + pp + 1 ≡ pp + 2 ≡ 2 ( mod p)
Töø ñieàâu kieän baøi toaùn suy ra: 2 ≡ 0 ( mod p )
Maâu thuaãn vì p > 2 .
Vaäy (p , q) = (2 , 5) hoaëc (5 , 2) laø hai caëp soá nguyeân toá
caàn tìm.

Baøi 5:
Goïi a,b,c laàn löôït laø ñoä daøi 3 caïnh tam giaùc BC, CA, AB
D laø giao ñieåm cuûa BC vôùi phaân giaùc trong goùc A
cuûa tam giaùc ABC
E, F laàn löôït laø hình chieáu cuûa D leân AB, AC
Ta coù ñoä daøi phaân giaùc AD:
AD = [[bc.(b+c-a).(a+b+c)]/ [ (b+c) 2]]1/2
= [bc.(b+c-a). l ]1/2/ (b+c)
Töù giaùc AEDF noäi tieáp ñöôøng troøn ñöôøng kính AD, ta coù
lA = EF = AD sin(A) = (2.AD. T)/(b.c)
Vôùi T laø dieän tích tam giaùc ABC
lA = [2.T. ((b+c-a).l)1/2]/ [(b+c). (bc)1/2].
Töông töï ta coù
lB = [2.T. ((c+a-b).l)1/2]/ [(c+a). (ca)1/2].
lC = [2.T. ((a+b-c).l)1/2]/ [(a+b). (ab)1/2].
Suy ra:
lA. lB lC = [8.T3.l. [(a+b-c).(b+c-a).(c+a-b).l]1/2]/[8.a.b.c.(a+b).
(b+c).(c+a)]
Söû duïng coâng thöùc Heron ta coù
lA. lB lC = [l3. (a+b-c)2. (b+c-a)2 . (c+a-b)2]/[ 8.a.b.c.(a+b).
(b+c).(c+a)]
≤ [l3. (a+b-c)2. (b+c-a)2 . (c+a-b)2]/ [64.a2.b2.c2]
Ta coù: (a+b-c).(a+c-b) = a2 – (b-c)2 ≤ a2
(a+b-c).(b+c-a) = b2 – (c-a)2 ≤ b2
(c+a-b).(c+b-a) = c2 – (a-b)2 ≤ c2
Töø ñoù suy ra:
lA. lB. lC ≥ (1/64). l3

Baøi 6:
a) Cho a, b laø hai soá nguyeân döông, ta coù
(a+b).(f(a)+f(b)) = (a+b) . f(a)+ (a+b). f(b)
≤ f(a.(a+b)) + f(b.(a+b))
≤ a.f(a+b)+a-1+b.f(a+b)+b-1
<(a+b).(f(a+b)+1)
Suy ra f(a)+ f(b) < f(a+b) +1
Vì f nhaän giaù trò nguyeân neân f(a)+ f(b) ≤ f(a+b)
Töông töï,
(a+b).f(a+b) = a.f(a+b) + b.f(a+b)
≤ f(a.(a+b)) + f(b.(a+b))
≤ (a+b).f(a)+ a+b-1+ (a+b).f(b)+ a+b-1
< (a+b).(f(a)+f(b)+2)
Suy ra f(a+b) < f(a)+f(b)+2
Vì f nhaän giaù trò nguyeân neân f(a+b) ≤ f(a)+f(b)+1
Vaây ta coù: f(a)+f(b) ≤ f(a+b) ≤ f(a)+f(b)+1
c)
Xeùt taäp hôïp sau
S= {f(2007.n) – 2007.f(n) / n= 1,2, …, 2007}
Vì 2007.f(n) ≤ f(2007.n) ≤ f(2007.n) +2005 neân taäp hôïp S coù
nhieàu nhaát laø 2006 phaân töû. Theo nguyeân lí Dirichlet toàn
taïi hai soá nguyeân döông khaùc nhau k, m ≤ 2007 sao cho:
f(2007.k) – 2007.f(k) = f(2007.m) – 2007.f(m),
do ñoù
f(2007.k) – f(2007.m) = 2007 . (f(k) –f(m))
Khoâng maát tính toång quaùt, ta giaû söû k ≥ m. Ñaët k = m+c
Baát phöông trinh töông ñöông
f(2007.(m+c)) – f(2007.m) = 2007 . (f(m+c) – f(m) (*)
Maët khaùc ta coù
f(2007.c) ≤ f(2007.(m+c) – f(2007.m) ≤ f(2007.c)+1
f(c) ≤ f(m+c) – f(m) ≤ f(c) + 1
Xeùt 4 tröôøng hôïp keát hôïp vôùi (*) ta suy ra: f(2007.c) =
2007.f(c).

Iranian Mathematical Olympiad (2006-2007)

Voøng 1
Baøi 1: Vôùi moãi m,n >2 laø soá nguyeân chöùng minh raèng
toàn taïi moät daõy soá a 0, a1,…, ak nguyeân sao cho a0=m, ak=n
vaø
ai + ai+1 laø öôùc soá cuûa ai.ai+1 + 1 vôùi i= 0,1, …, k-1

Baøi 2: Cho I1, I2, …, In laø n khoaûng ñoùng trong R sao cho
trong soá baát kì k taäp cuûa chuùng coù 2 taäp coù phaàn giao
khaùc roãng. Chöùng minh raèng coù theå choïn k-1 ñieåm trong
R sao cho baát kì khoaûng ñoùng naøo cuõng chöùa moät ñieåm
trong nhöõng ñieåm ñaõ choïn.

Baøi 3: Cho A, B, C, D laø 4 ñieåm naèm treân ñöôøng troøn Ω


theo thöù töï. Chöùng minh raèng toàn taïi 4 ñieåm M 1, M2, M3, M4
treân ñöôøng troøn sao cho 4 ñieåm ñoù taïo thaønh moät töù
giaùc coù 2 ñöôøng cheùo vuoâng goùc nhau, ñoàng thôøi thoûa
maõn:
AMi / BMi = DMi / CMi vôùi i = 1,2,3,4.

Baøi 4: Tìm taát caû ña thöùc p(x, y) trong R[x,y] sao cho vôùi
moïi x,y thuoäc R ta coù:
p(x+y, x-y) = 2.p(x, y)

Baøi 5: Cho hai ñöôøng troøn C1 vaø C2 sao cho taâm cuûa
ñöôøng troøn C1 naèm treân ñöôøng troøn C2. Neáu M vaø N laø
giao ñieåm cuûa hai ñöôøng troøn, AB laø ñöôøng kính baát kì
cuûa C1, A1 vaø B1 laàn löôït laø giao ñieåm cuûa AM vaø BN vôùi
ñöôøng troøn C2. Chöùng minh raèng A1B1 coù ñoä daøi baèng
baùn kính cuûa ñöôøng troøn C1.

Baøi 6: Chuùng ta coù moät giaù saùch cuûa n cuoán saùch


ñöôïc xeáp vôùi nhau, vaø ñöôïc ñaùnh soá 1,2,…,n . Trong moãi
löôït di chuyeån ta laøm n söï di chuyeån theo kieåu sau. Trong
laàn di chuyeån thöù i cuûa moãi löôït, thay ñoåi vò trí i cuoán
saùch ñaàu tieân, chæ trong nhoùm ñoù. Sau moãi laàn löôït di
chuyeån ta baét ñaàu moät löôït môùi töông töï nhö löôït tröôùc
( nghóa laø caùc cuoán saùch thay ñoåi vò trí theo qui luaät
voøng tröôùc) . Chöùng minh raèng sau moät soá di chuyeån, ta
seõ tìm laïi söï saép xeáp ñaàu tieân.

Voøng 2
Baøi 1: Moät ña dieän ñeàu laø moät ña dieän loài vaø taát caû
caùc maët cuûa noù ñeàu laø ña giaùc ñeàu. Ta goïi moät ña
dieän ñeàu laø TLP, neáu vaøo chæ neáu khoâng coù maët naøo
cuûa noù laø moät tam giaùc
a) Chöùng minh raèng moãi TLP noäi tieáp trong moät hình
caàu.
b) Chöùng minh raèng caùc maët cuûa TLP laø nhöõng ña
giaùc thuoäc nhieàu nhaát 3 loaïi ña giaùc ( nghóa laø coù
moät taäp hôp  m, n, p sao cho moãi maët cuûa TLP laø moät
n-giaùc ñeàu hoaëc m-giaùc ñeàu hoaëc p-giaùc ñeàu ).
c) Chöùng minh raèng chæ coù moät TLP vôùi caùc maët chæ
laø nguõ giaùc vaø luïc giaùc.
d) Cho n > 3, hình laêng truï caùi maø caùc maët cuûa noù
coù hai n-giaùc ñeàu vaø n hình vuoâng laø moät TLP.
Chöùng minh raèng ngoaøi nhöõng TLP naøy coù voâ soá
TLP khaùc.

Baøi 2: Moät chaát löu ñang chaûy trong moät oáng daãn
thaúng. Vôùi moãi phaân töû cuûa löu chaát, neáu noù ôû taïi
moät ñieåm vôùi toïa ñoä x, sau thôøi gian t giaây noù seõ ôû vò
trí P (t,x). Chöùng minh raèng neáu P(t,x) laø moät ña thöùc thì
taát caû caùc phaân töû ñang di chuyeån vôùi duy nhaát moät
vaân toác coá ñònh.

Baøi 3: Giaû söû raèng C laø moät taäp con loài trong R 3 vôùi
theå tích döông. Giaû söû C 1, C2, …,Cn laø n hình gioáng C
chuyeån thaønh ( khoâng quay) sao cho C i ∩ C khaùc roãng vaø
Ci vaø Cj giao nhau nhieàu nhaát ôû bieân, vôùi i  j. Chöùng minh
raèng n ≤ 27 vaø 27 laø giaù trò lôùn nhaát.
a) Chöùng minh cho tröôøng hôïp C ñoái xöùng.
b) Chöùng minh cho C baát kì.

Baøi 4: Cho moät soá höõu haïn hình khoâng giao nhau trong
maët phaúng. Moät söï phaân chia loài laø moät söï phaân chia
cuûa maët phaúng thaønh nhöõng phaàn loài, ñeå maø moãi
phaàn chöùa ñuùng moät trong nhöõng hình treân. Nhöõng
phaàn coù theå giao nhau chæ taïi bieân vaø chuùng phaûi phuû
maët phaúng.
Cho moãi taäp hôïp sau, toàn taïi hay khoâng moät söï phaân
chia loài ?
a) Höõu haïn nhöõng ñieåm rôøi nhau.
b) Höõu haïn nhöõng ñoaïn thaúng khoâng caét nhau.
c) Höõu haïn nhöõng ñóa troøn khoâng caét nhau.

Baøi 5: Cho A   vaø a, b   , ta ñònh nghóa:


a. A  b   ax  b / x  A
Neáu a  0 thì ta noù a.A+b ñoàng daïng A. Vôùi taäp Cantor C
laø taäp nhöõng soá nguyeân khoâng aâm maø khoâng coù soá 1
thuoäc trong söï bieåu dieãn côû sôû-3.
Ví duï C = (3C) U (3C+2) hoaëc C = (3C) U (9C+6) U
(3C+2)
Moät söï bieåu dieãn cuûa C laø moät söï phaân chia cuûa C
thaønh moät höõu haïn (lôùn hôn 1) cuûa nhöõng taäp hôïp
ñoàng daïng vôùi C, nghóa laø :
C  U in1 C i
Vôùi Ci = ai C + bi laø moät taäp ñoàng daïng vôùi C. Ta goïi moät
söï bieåu dieãn cuûa C laø moät söï bieåu dieãn nguyeân thuûy,
neáu vaø chæ neáu hoäi cuûa moät soá Ci (khoâng laø taát caû)
khoâng ñoàng daïng vôùi C vaø khoâng baèng C.
Xeùt moät ñaïi dieän nguyeân thuûy taäp Cantor. Chöùng minh
raèng:
a) ai >1
b) ai laø moât luõy thöøa cuûa 3.
c) ai > bi
d) Chæ coù moät söï phaân chia nguyeân thuûy cuûa C laø : C
= (3C) U (3C+2)

Baøi 6: Cho P(x) vaø R(x) laø hai ña thöùc coù heä soá nguyeân,
P(x) laø ña thöùc baäc leû. Chöùng minh raèng toàn taïi ña thöùc
Q(x) laø moät ña thöùc baäc leû vôùi heä soá nguyeân sao cho:
P(x) | Q(R(x)).

Voøng 3

Baøi 1: Cho A laø taäp con lôùn nhaát cuûa taäp 1,2,..., n sao
cho moãi phaàn töû cuûa A chia heát cho nhieàu nhaát moät
phaàn töû khaùc cuûa A. Chöùng minh raèng:
2.n n
 | A |  3.  .
3 4
Baøi 2: Toàn taïi hay khoâng moät daõy soá nguyeân döông a 0,
n

a1, …, an sao cho vôùi moãi i  j, (ai , aj) = 1 vaø ña thöùc  a .x


1
i
i

laø ña thöùc baát khaû qui trong Z[x].


Baøi 3: Cho tam giaùc ABC caân ( AB = AC ). Töø A, ta veõ moät
ñöôøng thaúng l song song vôùi BC. Hai ñieåm P, Q laàn löôït
naèm treân ñöôøng trung tröïc cuûa AB, AC, sao cho PQ vuoâng
goùc vôùi BC. M, N laø hai ñieåm naèm treân l sao cho APM vaø
AQN vuoâng. Chöùng minh raèng :
1 1 2
 
AM AN AB

Baøi 4: Giaû söû n ñöôøng thaúng naèm trong maët phaúng, sao
cho khoâng coù 2 ñöôøng thaúng naøo song song vaø khoâng
coù 3 ñöôøng naøo ñoàng qui. Goùc cuûa 2 ñöôøng thaúng ñònh
nghóa laø goùc nhoû nhaát taïi ñieåm giao cuûa chuùng. Tìm
giaù trò lôùn nhaát cuûa toång C n2 goùc ñöôïc taïo thaønh töø 2
ñöôøng thaúng trong n ñöôøng thaúng.
Baøi 5: Cho O naèm trong tam giaùc ABC sao cho OA = OB + OC
. Laáy B’, C’ laàn löôït laø troïng taâm cuûa tam giaùc AOC vaø
AOB. Chöùng minh raèng ñöôøng troøn ngoaïi tieáp cuûa COC ’
vaø BOB’ tieáp xuùc nhau.

Baøi 6: Tìm taát caû caùc ña thöùc P baäc 3 sao cho vôùi moãi
soá thöïc khoâng aâm ta coù:
P (x+y) ≥ P(x) + P(y) .
Phụ lục 1:

BỘ GIÁO DỤC VÀ ĐÀO


TẠO
TRƯỜNG ĐẠI HỌC FPT
----------***----------

HƯỚNG DẪN LÀM BÀI THI TUYỂN SINH


Bài thi dưới dạng trắc nghiệm, mỗi câu đúng được 1 điểm, câu sai không bị trừ
điểm bao gồm 95 câu chia làm 3 phần. Thời gian thi là 120 phút.
Phần 1: Kỹ năng tính toán, 20 câu
Phần 2: Kỹ năng phân tích, xử lý tính đầy đủ của thông tin để giải quyết bài toán,
25 câu
Phần 3: Lập luận logic, 50 câu

Hướng dẫn chi tiết từng phần:

Phần 1:
Phần này gồm 20 câu hỏi nhằm mục đích kiểm tra kỹ năng làm việc với các con số
(kỹ năng tính toán) của thí sinh.

Ví dụ:
Trong các năm 1997, 1998, 1999 lương của An được tăng 10% mỗi năm. Hỏi
lương của An năm 1999 đã tăng bao nhiêu phần trăm so với năm 1997?
(A)10%
(B) 11%
(C) 20%
(D)21%
(E) 30%

Giải: Giả thiết L là lương của An năm 1997. Năm 1998 lương của An là 110% của
L hay là (1.1)L và năm 1999 sẽ là 110% của (1.1)L = (1.1)(1.1)L = 1.21 L. Như
vậy lương của An năm 1999 đã tăng 21% so với năm 1997. Đáp án (D)

Phần 2:
Phần này gồm 25 câu và 5 phương án trả lời cho trước (A,B,C,D,E) chung cho tất
cả các câu. Mỗi câu hỏi sẽ có 2 dữ kiện đi kèm (1) và (2). Nhiệm vụ của thí sinh là
tìm ra phương án đúng (trong 5 phương án trả lời cho trước) cho mỗi câu hỏi.
5 phương án trả lời cho trước như sau:
(A) Dùng một mình dữ kiện (1) là đủ để có thể trả lời câu hỏi, nhưng dùng một
mình dữ kiện (2) thì không đủ.
(B) Dùng một mình dữ kiện (2) là đủ để có thể trả lời câu hỏi, nhưng dùng một
mình dữ kiện (1) thì không đủ.
(C) Phải dùng cả 2 dữ kiện (1) và (2) mới trả lời được câu hỏi, tách riêng từng
dữ kiện sẽ không trả lời được.
(D) Chỉ cần dùng một dữ kiện bất kỳ trong 2 dữ kiện đã cho cũng đủ để trả lời
được câu hỏi.
(E) Dùng cả 2 dữ kiện đã cho cũng không thể trả lời được câu hỏi.

Ví dụ 1:
Cho biết k là một số nguyên. Hỏi k có chia hết cho 12 không?
(1) k chia hết cho 4
(2) k chia hết cho 3

Giải: Riêng dữ kiện (1) là không đủ để trả lời, chẳng hạn 24 và 16 đều chia hết
cho 4 nhưng 24 thì chia hết cho 12 còn 16 thì không. Riêng dữ kiện (2) cũng
không đủ để trả lời, chẳng hạn 24 và 15 đều chia hết cho 3 nhưng 24 thì chia hết
cho 12 còn 15 thì không.
Từ dữ kiện (1) suy ra k = 4m với m là một số nguyên. Giả thiết dữ kiện (2) cũng
được đáp ứng, khi đó 4m sẽ chia hết cho3. Do 4 không chia hết cho 3 nên m sẽ
phải chia hết cho 3. Từ đây ta suy ra m = 3n với n là một số nguyên => k = 4 x 3n
= 12n , chia hết cho 12. Như vậy dùng cả 2 dữ kiện (1) và (2) sẽ trả lời được câu
hỏi đặt ra ở trên. Đáp án (C)

Ví dụ 2:
x có phải là một số chẵn (Giả thiết n và p là các số nguyên)?
(1) x = (n + p)2
(2) x = 2n + 10p

Giải: Dễ dàng nhận thấy từ dữ kiện (2) sẽ suy ra được x là số chẵn vì 2n + 10p =
2(n + 5p) là một số chia hết cho 2. Dữ kiện (1) là không đủ để trả lời câu hỏi x có
phải là số chẵn hay không, chẳng hạn (1 + 3) 2 = 16 là số chẵn, còn (2 + 3) 2 = 25 lại
là số lẻ. Như vậy đáp án sẽ là (B).

Phần 3:
Phần này gồm 50 câu hỏi nhằm mục đích kiểm tra năng lực phân tích của thí sinh.
Có 2 dạng câu hỏi cơ bản và cả 2 đều đỏi hỏi khả năng lập luận mạnh: lập luận
phân tích (khoảng 38 câu) và lập luận logic (khoảng 12 câu). Mỗi câu hỏi hoặc
nhóm câu hỏi sẽ dựa trên một bài toán tình huống hoặc một nhóm các điều kiện.
Việc vẽ bảng hoặc biểu đồ sẽ rất có ích cho thí sinh trong việc trả lời một số câu
hỏi.
Ví dụ 1 (Câu hỏi 1 - 4)
Có 6 nhà ngoại giao A, B, C, D, E, F ngồi với nhau. Không phải tất cả đều nói
cùng một ngôn ngữ nhưng mỗi ngôn ngữ đều có đủ số người biết để họ có thể dịch
lẫn cho nhau.
- A và D chỉ nói được tiếng Anh, tiếng Pháp và tiếng Ý
- B chỉ nói được tiếng Anh, tiếng Pháp và tiếng Nga
- C chỉ nói được tiếng Đức và tiếng Ý
- E chỉ nói được tiếng Ý
- F chỉ nói được tiếng Nga

Câu 1: Ngôn ngữ nào được nhiều người nói nhất?


(A) tiếng Anh
(B) tiếng Pháp
(C) tiếng Đức
(D) tiếng Ý
(E) tiếng Nga

Câu 2: Cặp nào sau đây có thể nói chuyện không cần phiên dịch?
(A) A và F
(B) B và C
(C) B và E
(D) E và F
(E) B và F

Câu 3: Ai có thể làm phiên dịch cho B và C?


I. A
II. D
III. E
IV. F

(A)Chỉ I
(B) I và II
(C) I, II và III
(D)II, III và IV
(E) I, II và IV

Câu 4. Nếu C và F muốn nói chuyện với nhau thì họ sẽ cần ít nhất mấy phiên
dịch?
(A)0
(B) 1
(C) 2
(D)3
(E) 4

Giải: Ta vẽ bảng sau:

tiếng Anh tiếng Pháp tiếng Ý tiếng Nga tiếng Đức


A Có Có Có Không Không
B Có Có Không Có Không
C Không Không Có Không Có
D Có Có Có Không Không
E Không Không Có Không Không
F Không Không Không Có Không

Câu 1: Dễ dàng nhận thấy tiếng Ý có nhiều người nói nhất (4: A, C, D, E). Đáp án
(D)

Câu 2: Chỉ có 1 trong 5 cặp được liệt kê là có chung ngôn ngữ, đó là B và F (cùng
nói tiếng Nga). Đáp án (E)

Câu 3: Cả A và D đếu có thể nói chuyện bằng tiếng Anh với B, sau đó dịch sang
tiếng Ý cho C. E và B không có ngôn ngữ nào chung, vì vậy E không thể làm
phiên dịch cho B. Tương tự F và C cũng không có ngôn ngữ nào chung, vì vậy F
không thể làm phiên dịch cho C. Như vậy đáp án đúng là (B)

Câu 4: Người duy nhất có thể nói chuyện trực tiếp với F là B (bằng tiếng Nga),
như vậy cần ít nhất là 1 phiên dịch cho C và F. Tuy nhiên B và C không thể nói
chuyện trực tiếp với nhau, bởi vậy ta cần ít nhất thêm một phiên dịch nữa. Trong
lời giải câu hỏi trước (câu 3) ta đã biết rằng B và C chỉ cần 1 phiên dịch (A hoặc
D). Như vậy số phiên dịch ít nhất cần để cho C và F nói chuyện được với nhau là
2. Đáp án (C)

Ví dụ 2:
“Nếu Bình là thành viên của đội tuyển bóng đá thì Bình cũng là thành viên của đội
tuyển bóng chuyền”. Phát biểu trên được rút ra bằng cách suy luận logic từ phát
biểu nào sau đây?
(A)Tất cả các thành viên của đội tuyển bóng chuyền là thành viên của đội
tuyển bóng đá
(B) Bình là thành viên của đội tuyển bóng chuyền hoặc đội tuyển bóng đá.
(C) Người nào là thành viên của đội tuyển bóng đá thì cũng là thành viên của
đội tuyển bóng chuyền.
(D)Một vài thành viên của đội tuyển bóng chuyền cũng là thành viên của đội
tuyển bóng đá.
(E) Bình là thành viên của đội tuyển bóng đá.

Giải: (B) và (E) không liên quan trực tiếp đến phát biểu trên. Từ (A) không thể
suy ra phát biểu trong đề bài (chẳng hạn đội bóng chuyền có 6 người A1, A2, A3,
A4, A5, A6 - tất cả đều là thành viên của đội bóng đá gồm 7 người A1, A2, A3,
A4, A5, A6 và Bình. Như vậy Bình là thành viên của đội tuyển bóng đá nhưng
không phải là thành viên của đội tuyển bóng chuyền). Từ (D) rõ ràng cũng không
suy ra được phát biểu trong đề bài. Đáp án (C): Phát biểu đúng cho mọi thành viên
của đội tuyển bóng đá, tức là cũng đúng cho Bình nếu Bình là thành viên của đội
tuyển bóng đá.
Phụ lục 2:

Dự thảo kế hoạch Cuộc thi Olympic Toán


dành cho học sinh phổ thông do đại học FPT tổ chức
Mục đích.
+ Tìm kiếm và khuyến khích các học sinh có năng khiếu về toán. Thông qua nội
dung thi, phát triển tư duy logic, tư duy thuật toán, khả năng phân tích và tổng
hợp, khả năng làm việc độc lập và làm việc nhóm, giới thiệu với các học sinh về
các ứng dụng của toán học trong cuộc sống và trong các khoa học khác, đặc biệt là
ứng dụng của Toán học trong CNTT và của CNTT trong toán học.
+ Các học sinh có năng khiếu sẽ tiếp tục được bồi dưỡng thông qua các hình thức
- Đào tạo từ xa (mô hình hàm thụ)
- CLB tại FPT University
+ Các học sinh đoạt giải sẽ được giải thưởng dưới các hình thức: tiền mặt, tài liệu
học tập, học bổng FU.

Đối tượng.
+ Các học sinh phổ thông trung học yêu thích toán
+ Sinh viên đại học năm 1, 2, sinh viên Aptech, Arena (Optional).

Hình thức.

Cuộc thi sẽ được tổ chức thành nhiều vòng. Các vòng sẽ được tổ chức ở đâu, như
thế nào (qua Internet, tại trường, tập trung) thì sẽ bàn kỹ sau. Riêng vòng cuối sẽ
tổ chức tại trường, có thể có quay truyền hình vì rất vui.
+ Vòng sơ tuyển: Thi trắc nghiệm
Đề bài gồm 50 câu hỏi trắc nghiệm để kiểm tra các kiến thức toán cơ bản, kỹ năng
tính toán, tư duy logic, khả năng phân tích và tổng hợp thông tin.

Sau đây là ví dụ một số câu hỏi:

1. Có 3 chiếc đồng hồ cũ chỉ còn kim phút. Cả 3 đồng hồ đều chạy nhanh. Các
đồng hồ A, B và C sau mỗi tiếng chạy nhanh tương ứng là 2, 4 và 6 phút tương
ứng. Vào lúc 12 giờ đêm, cả 3 kim phút đều chỉ vào số 12. Hỏi sau bao nhiêu giờ,
các kim phút này lại cùng gặp nhau, tức là chỉ đúng số phút như nhau?
A) 20 B) 25 C) 30 D) 35 E) 40

2. Nếu bán kính của hình cầu tăng lên 5% thì thể tích hình cầu tăng lên bao nhiêu
phần trăm (chính xác đến chữ số hàng đơn vị)
A) 1.2% B) 10.5% C) 11.6% D) 15.8% E) 5.0%

3. 9 hình lập phương màu trắng và 18 hình lập phương màu đen có cạnh bằng 1
được dùng để xếp thành 1 hình lập phương lớn có cạnh bằng 3. Hỏi tỷ lệ phần diện
bề mặt xung quanh có màu trắng lớn nhất có thể bằng bao nhiêu?
A) 1/2 B) 13/27 C) 25/54 D) 4/9 E) 1/3

4. Tứ giác ABCD ngoại tiếp một đường tròn. Nếu AB = 4, BC = 5, CD = 3 thì DA


=?
A) 1 B) 2 3) 2.4 D) 3 E) 3.75

+ Vòng sơ khảo: Bài tập có đáp số


Đề bài gồm 20 câu hỏi để kiểm tra khả năng tính toán, tư duy thuật toán, khả năng
sử dụng máy tính bỏ túi, khả năng áp dụng các định lý toán học trong tính toán.
Thí sinh chỉ cần ghi đáp số, không cần giải thích. Nếu không nói gì thêm, đáp số
luôn là các số nguyên.

Sau đây là ví dụ 1 số câu hỏi:


1) Hãy tính tổng lập phương của 100 số nguyên dương đầu tiên.
2) Tứ giác ABCD có AB = 3, BC = 4, CD = 12, DA = 13 và góc ABC bằng
900. Hãy tính diện tích tứ giác.
3) Tìm 25 chữ số tận cùng của số 100!
4) Tính đến hết tháng 6 năm 2007, công ty FPT Software có 1999 nhân viên
và tuổi trung bình là 25.5. Trong tháng 7, công ty đã tuyển mới 235 nhân
viên mà tuổi trung bình mới của công ty FPT Software là 25.2. Hỏi tuổi
trung bình của các nhân viên tuyển mới là bao nhiêu?
(Ghi chú: Đáp số có thể có thể được làm tròn đến 2 chữ số sau dấu phẩy)
5) Trong một cuộc họp báo có một số nhà báo tham dự. Đầu tiên, tất cả các
nhà báo đều bắt tay nhau. Sau đó tổng thống Putin xuất hiện, ông bắt tay
với một số nhà báo mà ông quen. Người ta đếm được là có tất cả 100 cái
bắt tay. Hỏi có bao nhiêu nhà báo tham dự cuộc họp báo.
6) Trong một năm, ngoài các ngày chủ nhật, một nhân viên làm trong các cơ
quan nhà nước còn được nghỉ vào các ngày 1/1, 4 ngày Tết âm lịch, ngày
10/3 âm lịch (Giỗ tổ Hùng Vương), ngày 30/4 (giải phóng miền Nam), ngày
1/5 (Quốc tế lao động), ngày 2/9 (Quốc khánh). Giả sử rằng nếu ngày lễ
trùng vào ngày chủ nhật thì không được nghỉ bù. Hỏi trong một năm, nhân
viên cơ quan nhà nước có thể có nhiều nhất bao nhiêu ngày nghỉ? Ít nhất
bao nhiêu ngày nghỉ?
7) Cho a + b + c = 0, abc = 667. Hãy tính a3 + b3 + c3.

+ Vòng chung khảo:


Đây là vòng thi đồng đội, mỗi đội gồm 5 học sinh. Đề bài là một số bài toán mà
lời giải của nó đòi hỏi sự phối hợp giữa các thành viên, sử dụng các kỹ năng tổng
hợp, trong đó có cả kỹ năng dùng Excel, lập trình Pascal, kiến thức về thuật toán,
lý thuyết trò chơi.
Kết thúc vòng sơ khảo, danh sách các thí sinh lọt vào vòng chung khảo sẽ được
niêm yết (cùng địa chỉ email, số điện thoại, địa chỉ), các bạn sẽ cần tự liên hệ với
nhau để thành lập đội, đăng ký và cùng chuẩn bị, qua đó rèn luyện kỹ năng trao
đổi thông tin, kỹ năng làm việc nhóm.

Sau đây là ví dụ một số đề thi vòng chung khảo (Các trò này có thể được thể hiện
dưới dạng trò chơi đối kháng):

1. Trò chơi đong sữa: Có 3 bình dung tích m, n, k với m + n = k và (m, n) = 1. m, n


là các số lẻ. Ban đầu bình k lít đựng đầy sữa. Hãy tìm cách đong được k/2 lít.

2. Trò chơi qua cầu bằng đèn pin: Có 4 người cần qua cầu. Họ có thể đi với vận
tốc v1, v2, v3, v4 m/s. Vì trời tối nên qua cầu phải có đèn pin. Đèn pin không thể
chiếu sáng từ xa. Và mỗi lần chỉ có tối đa 2 người qua cầu. Hãy tìm phương án để
đưa cả 4 người qua cầu trong khoảng thời gian ngắn nhất.

3. Bài toán lạc đà và chuối: Một người cưỡi lạc đà từ A đến B. Khoảng cách AB
bằng s. Con lạc đà có thể cõng người này cùng k quả chuối, cứ 1 km nó cần ăn 1
quả chuối. Ở đầu A có sẵn N quả chuối, hỏi người đó có thể đem nhiều nhất bao
nhiêu quả chuối sang đầu B. (Ví dụ N = 300, s = 100, k = 100).

4. Trò chơi bốc sỏi: Có 3 đống sỏi có m, n, k viên sỏi tương ứng. Hai người chơi
trò chơi sau: Mỗi người, khi đến lượt của mình, chọn 1 trong các đống sỏi và bốc
đi một số lượng tuỳ ý các viên sỏi (ít nhất 1 viên). Ai đến lượt mình mà không còn
sỏi để bốc nữa là thua cuộc. Hãy tìm chiến thuật chơi để luôn thắng. (Tất nhiên là
phụ thuộc vào m, n, k)

5. Bài toán xếp lịch: Hãy xếp lịch thi đấu cho n đội bóng thi đấu vòng tròn 1 lượt,
thoả mãn các điều kiện sau:
1) Mỗi một lượt đấu, một đội đấu nhiều nhất 1 trận
2) Số lượt đấu là ít nhất

6. Khoảng cách giữa 2 điểm trên trái đất: Giả sử trái đất là một mặt cầu có bán
kính R = …. Km. Mặt trái đất được chia thành bán cầu Bắc và bán cầu Nam bởi
đường xích đạo. Các vĩ tuyến là các đường tròn song song với xích đạo, đánh số từ
00 (tại xích đạo) đến 900 (tại các cực). Mặt trái đất cũng được chia thành 2 nửa bán
cầu bởi kinh tuyến số 0, và chia thành 180 độ kinh đông và 180 độ kinh tây. Có hai
điểm A(x, y), B(z, t) trong đó x, y là các độ vĩ bắc, z, t là các độ kinh đông. Hãy
tìm công thức tính khoảng cách AB (trên mặt cầu).

7. Đường xoáy trôn ốc: Đánh số các điểm nguyên của mặt phẳng toạ độ theo hình
xoáy trôn ốc: (0, 0)  1, (0, 1)  2, (1, 1)  3, (1, 0)  4, (1, -1)  5, (0, -1) 
6, (-1, -1)  7, (-1, 0)  8, (-1, 1)  9, (-1, 2)  10, (0, 2)  11 …
1) Hãy xác định điểm (m, n) được đánh số mấy
2) Hãy xác định điểm đánh số N có toạ độ mấy

8. Đoán số: Đội A nghĩ ra một con số trong khoảng từ 1 đến 100. Đội B có thể hỏi
đội A một số câu hỏi có dạng: Số các bạn nghĩ có nằm trong tập X hay không?
Trong đó X là tập con nào đó của {1, 2, …, 100} mà đội B chọn (X thay đổi trong
mỗi câu hỏi). Đội A sẽ trả lời đúng hoặc sai. Với mỗi câu trả lời “Đúng”, đội B
phải trả 2.000 đồng, với mỗi câu trả lời “Sai”, đội B phải trả 1.000 đồng. Hãy tìm
chiến thuật cho đội B để đoán được số đội A nghĩ với tổng số tiền phải trả ít nhất.
Phụ lục 3:

Cờ carô: Những điều chưa biết?


Trần Nam Dũng

Bạn có biết chơi cờ ca-rô không? Nếu đặt một câu hỏi như vậy với một người Việt
Nam bất kỳ, đặc biệt là với giới trẻ, câu trả lời sẽ là có. Đây là một trò chơi đối
khác rất thông dụng trong giới học sinh, sinh viên, vì luật chơi rất đơn giản, dụng
cụ chơi cũng vô cùng rẻ tiền: một tờ giấy kẻ carô và 2 cây bút.

Thế nhưng ít ai biết rằng cờ ca-rô có một lịch sử lâu đời, có cả một lý thuyết chặt
chẽ, có Tạp chí chuyên đề, có những chương trình máy tính, có những giải vô địch
quốc gia và quốc tế và có cả Liên đoàn ca-rô thế giới.

Cờ carô là một trò chơi rất thông dụng trên toàn thế giới dưới những cái tên khác
nhau: cờ carô, gomoku, renju, nought and crosses, croix-zero, five-in-a-row,
connect5, itsutsu-ishi, gobang, piskvorky, kolko i kyzyk …

Luật chơi cũng có đôi chỗ khác nhau, nhưng luật cơ bản là hai bên thay phiên
nhau đi những nước đi (bằng các dấu X, O hoặc bằng các quân cờ đen, trắng), bên
nào có đường 5 quân liền nhau trên một hàng, một cột hoặc một đường chéo là
thắng.

Trong môn cờ carô, bên đi trước luôn có lợi thế rất lớn, thậm chí L.Victor Allis
còn chứng minh được rằng trong cờ carô tự do, bên đi trước luôn luôn thắng.
Chính vì vậy, người ta đưa ra một số luật bổ sung để hạn chế lợi thế của người đi
trước (quân đen) và tăng cường khả năng phòng thủ của người đi sau.

Chẳng hạn, trong luật chơi cờ ca-rô của Việt Nam (thông tin này có trên
wikipedia!), người nào đi được 5 quân nhưng đã bị chặn trước ở 2 đầu sẽ không
thắng. Một số nơi thì không công nhận 6 quân liền nhau là thắng, một số nơi lại
không công nhận bẫy 3 x 3 (tức là nước đi tạo thành 2 hàng 3 chưa có quân chặn
của đối phương sẽ bị cấm) …

Trong các luật bổ sung như vậy thì luật renju (theo tên gọi của người Nhật) là
phức tạp và chặt chẽ nhất, đồng thời cũng giúp cân bằng cơ hội của quân trắng (đi
sau) với quân đen. Xin giải thích ngắn gọn các luật cơ bản như sau: Ván cờ được
chơi trên bàn cờ 15 x 15 dòng kẻ. Hai bên sẽ thay phiên nhau đi những quân cờ
vào giao điểm của các hàng ngang và cột dọc. Người đi trước (quân đen) sẽ đi
quân đầu tiên vào tâm bàn cờ (giao của hàng 8, cột 8). Luật thắng thì cũng như cờ
ca-rô, tuy nhiên bên đen phải chịu những hạn chế cơ bản sau: không được tạo
thành một hàng 6 quân liên tiếp (overlines), không được tạo thành các bẫy 3x3
(double-threes) và 4x4 (double-fours) – nếu đi vào những nước như vậy coi như
thua. Cuối cùng, có luật về nước đi thứ 5 và thứ 7: Ở những nước thứ 5 và thứ 7,
quân đen phải đưa ra 2 phương án nước đi để quân trắng có quyền lựa chọn
(Alternative moves 5th and 7th). Hai lựa chọn này phải không đối xứng nhau (vì
nếu đối xứng thì coi như không có quyền chọn). Tất cả những luật này không
những hạn chế ưu thế của quân đen mà còn đem đến cho quân trắng những cơ hội
phải công. Ví dụ, quân trắng, bằng những nước đi của mình có thể buộc quân đen
tạo thành 1 hàng 6 quân liên tiếp hay tạo thành bẫy 3x3. Có những ván đấu kết
thúc ngay ở nước thứ 7 vì quân đen buộc phải đi 2 nước đối xứng.

Với những luật bổ sung như vậy, không như cờ carô tự do và các hình thức đơn
giản khác, renju đã được công nhận như một môn thể thao thực thụ và được phát
triển ở nhiều nước với các chuyên gia, nhà nghiên cứu, kỳ thủ và các hệ thống thi
đấu chặt chẽ. Liên đoàn Renju Nhật Bản (Nihon Renju Sha) được thành lập vào
năm 1966.

Ngày 8 tháng 8 năm 1988 Liên đoàn Renju thế giới (Renju International
Federation – RFI) được thành lập tại Stockholm, Thuỵ Điển. Một năm sau, giải Vô
địch thế giới về Renju được tổ chức tại Nhật Bản.

Các giải vô địch Renju thế giới những năm tiếp theo được tổ chức tại Moscow,
Nga (1991), Arjeplog, Thuỵ Điển (1993), Talinn, Estonia (1995), Saint Petersburg,
Nga (1997), Bắc Kinh, Trung Quốc (1999), Kyoto, Nhật Bản (2001), Vadstena,
Thuỵ Điển (2003), Talinn, Estonia (2005). Giải vô địch renju thế giới tiếp theo sẽ
được tổ chức trong năm nay tại Tyumen, Nga.
Chiếm ngôi quán quân ở giải Vô địch thế giới lần thứ nhất và lần thứ hai là
Shigeru Nakamura (Nhật Bản), lần thứ ba là Ando Meritee (Estonia). Đương kim
vô địch thế giới về renju hiện nay là Ando Meritee (Estonia).

Hiện nay Liên đoàn Renju thế giới có 10 thành viên chính thức là Nhật Bản, Nga,
Thuỵ Điển (3 thành viên sáng lập), Armenia, Azerbaijan, Trung Quốc, Đài Loan,
Estonia, Hàn Quốc, Uzebekistan. Các nước sau đây có những người đại diện,
nhưng chưa phải là thành viên chính thức: Bạch Nga, Estonia, Áo, Bulgaria,
Canada, Croatia, Đan Mạch, Phần Lan, Pháp, Đức, Israel, Ý, Mông cổ, Hà Lan,
Na Uy, Ba Lan, Rumani, Slovakia, Ukraina, Mỹ.

Chủ tịch Liên đoàn Renju thế giới hiện nay là ông Peter Jonsson (Thuỵ Điển), các
phó chủ tịch là các ông Norihoko Kawamura (Nhật Bản), Alexander Nosovsky
(Nga). Tổng thư ký RIF là ông Pavel Salnikov (Nga) và thủ quỹ là ông Stefan
Karlsson (Thuỵ Điển).

Ông Alexandr Nosovsky chính là một trong những người đã có công truyền bá
renju (với các quy tắc chặt chẽ của nó) vào các nước Liên Xô cũ. Nhận thấy luật
chơi renju còn quá phức tạp, khó nhớ và khó kiểm soát (cần phải có trọng tài với
sự hiểu biết luật vững vàng), vào những năm thập niên 80 của thế kỷ 20, chính
Nosovsky là người đã đưa luật chơi cho carô tự do (Five-in-a-row, hay Gomoku)
và có hàng loạt bài viết trên tạp chí Khoa học và đời sống (Nauka I zizn) để truyền
bá về cách chơi và luật của môn carô giản lược này. Cụ thể Nosovsky bỏ đi tất cả
các hạn chế mà trong renju truyền thống áp đặt lên quân đen. Bù vào đó, ông đưa
ra 1 luật duy nhất, đó là hạn chế khu vực nước đi thứ hai của quân đen (đi trước).
Cụ thể, quân đầu tiên quân đen phải đi vào giữa bàn cờ. Nước đi thứ hai quân đen
không được đi vào những vị trí thuộc hình vuông trung tâm (có tâm là tâm bàn cờ,
cạnh 4 x 4).

Với luật giản lược này, Gomoku dễ dàng thâm nhập giới sinh viên học sinh Nga,
các nước Liên Xô cũ và châu Âu hơn. Và trong 2 giải vô địch thế giới đầu tiên,
bên cạnh Renju truyền thống, giải vô địch thế giới về Gomoku cũng đã được tổ
chức song song. Tuy nhiên, do sau này người ta thấy những hạn chế trên của
Nosovsky đưa ra vẫn không hạn chế được ưu thế của quân đen (điều này phản ánh
qua kết quả các trận đấu – các trận đen thắng chiếm ưu thế tuyệt đối) nên RIF đã
không tổ chức các giải Gomoku trong những năm tiếp theo. Vì thế Iuri Tarannikov
(Nga), nhà vô địch Gomoku thế giới năm 1991 hiện vẫn là đương kim vô địch
Gomoku cho đến thời điểm này. Dù vậy, các giải Gomoku mở rộng, giải tầm quốc
gia và quốc tế vẫn được tổ chức đều đặn tại các nước thành viên RIF, đặc biệt là ở
châu Âu.

Cuối cùng, chúng tôi xin giới thiệu hình ảnh bàn cờ và kịch bản của một ván đầu
cờ carô trên bàn cờ chuẩn.
Bàn cờ có kích thước là 15x15. Các nước đi được đi vào giao điểm của hai đường
thẳng. Quân đầu tiên phải được đi vào chính giữa bàn cờ. Ván cờ được ghi lại
bằng các quân số ký hiệu thứ tự nước đi. Các quân đen có số thứ tự 1, 3, 5, 7 …
các quân trắng có thứ tự tương ứng là 2, 4, 6, 8 …

Để tìm hiểu thêm về cờ carô và các biến thể của nó như Renju, Gomoku, bạn đọc
có thể vào các trang web sau:

Cổng thông tin của RFI: www.renju.net


Trang web của Liên đoàn renju Thuỵ Điển: renju.se/rif
Wikipedia: en.wikipedia.org/wiki/Gomoku, en.wikipedia.org/wiki/Renju
Website dạy đánh cờ renju online: www.renjuclass.com
Phát triển gomoku, một chủ đề trong diễn đàn của dân game Việt Nam nói về
gomoku: http://www.gamevn.com/showthread.php?t=105121

You might also like